You are on page 1of 53

SAMPLE QUESTIONNAIRE FOR “ISMS” PHILOSOPHY OF EDUCATION to accept them or not. What kind of philosophy does he practice?

(SOCIAL DIMENSIONS) A. Idealism C. Humanism


B. Essentialism D. Existentialism
1. The Philippine Elementary School Curriculum gives greater emphasis on
the development of basic skills like reading, writing, and arithmetic. What is RATIONALIZATION: D - Existentialism. Existentialism is a philosophy that
the philosophical basis for this? emphasizes subjectivity, freedom and responsibility.
A. Pragmatism C. Essentialism
B. Perennialism D. Existentialism 5. When a teacher emphasizes that man’s sense should be trusted because
they are the only
RATIONALIZATION: C. Essentialism. Essentialism - is an educational theory way to acquire knowledge, the teacher can be regarded as .
rooted in classical realism and idealism which advocates curricular reform
stressing the essentials of the basics like the 3R's. A. Naturalist C. Empiricist
Pragmatism- is regarded as an American philosophy whose various forms B. Realist D. Pragmatist
advocate experimentalism,
RATIONALIZATION: C -Empiricism. Empiricism upholds that the only
instrumentalism, functionalism, and practicalism and their curricular
source of knowledge is the senses and sense-based experience.
offerings imply a wide range of subject areas.
Pereninialism- is a traditional educational theory that puts premium on 6. Teacher K views her pupils as unique, free choosing, and responsible
eternal truth as contained in the "Great Books". individuals. She plans activities where the pupil can develop his unique
Existentialism - personalizes knowledge to the individual. The person personality. What theory underlies this nature of the pupil?
chooses the knowledge that he deems is relevant to his process of becoming A. Realism C. Existentialism
to realize his essence. B. Essentialism D. Progressivism
2. Teacher H asks one of her students, “What do you want to become when RATIONALIZATION: C. Existentialism. Existentialism is the philosophy of
you grow up?” This subjectivity of self hood whose fundamental doctrine proclaims man’s
question is an indication of what kind of philosophy? A. Progressivism C. freedom in the accomplishment of his destiny. In this case, emphasis is
Existentialism given on the students rather than on the curriculum content.
B. Naturalism D. Idealism
7. "Learning is the process of retrieving prior learning", this is a statement
RATIONALIZATION: D - Idealism. Idealism is the answer because it stresses from _. A. Constructivist C. Progressivist
the existence of ideas independent from the material world. Ideas that B. Reconstructivist D. Empiricist
which exist in the mind are the only reality.
RATIONALIZATION: A. Constructivist. As defined, Constructivism is a
3. Teacher X has not only explained the concept of Philosophy of Education philosophy of learning which asserts that reality does not exist outside of
but also imparted this to her students. This demonstrates what kind of the human conceptions. It is the individual that construct reality by
philosophy? reflecting on his own experience and gives meaning to it. It is founded on
A. Naturalism C. Realism the premise that by reflecting on our own experiences, we construct our
B. Idealism D. Perennialism own understanding of the world we live in. Therefore, learning is simply the
process of adjusting our mental modes to accommodate new experiences.
RATIONALIZATION: C - Realism. Realism concerns with what is real, actual.
For ideas to be realized, they must be transferred or demonstrated. 8. Teacher U teaches to his pupils that pleasure is not the highest good.
Teacher's teaching is against what philosophy?
4. In his class, Teacher M always presents principles and values so as to
A. Realism C. Epicureanism
encourage his students to examine them and decide for themselves whether
B. Hedonism D. Empiricism
1
RATIONALIZATION: B. Hedonism. Epicureanism is just a form of Hedonism. 13. Matilda is an advocate of the principle “making the most of your life”.
Hedonism is the general, Epicureanism is the specific. Always select the She is indeed an
general answer. advocate of what kind of philosophy?
A. Humanism C. Realism
9. After listening to the homily of the Priest about fidelity, Catherine has a B. Perennialism D. Existentialism
moment of reflection. Her understanding of the value of fidelity has become
deeper as she related this to her past experience. This typifies what kind of RATIONALIZATION: A. Humanism. Humanism is a philosophy that stresses
philosophy? to “live life to the fullest”.
A. Constructivism C. Humanism
B. Reconstructivism D. Existentialism 14. Teacher X is a very dedicated teacher in the nursery. Her foremost
concern is for students to learn how to adapt themselves in the
RATIONALIZATION: A. Constructivism. Constructivism is a philosophy of environment. This shows that Teacher X upholds what kind of philosophy?
learning which asserts that reality does not exist outside the human A. Naturalism C. Progressivism
conceptions. It is the individual who construct reality by reflecting on his B. Pragmatism D. Rationalism
own experience and gives meaning to it.
RATIONALIZATION: A. Naturalism. Naturalism aims to unfold the child’s
10. After studying the Principle of Identity, Teacher W asks her students to potential, not to prepare him for a definite vocation or social position but to
determine which among the given set of problems conforms to the said prepare him to adapt to the changing times and needs.
identity. This shows that Teacher W upholds what kind of Philosophy?
A. Perennialism C. Essentialism 15. Teacher J serves as an inspiration to his students because of his
B. Progressivism D. Naturalism efficiency and effectiveness as a teacher. The mind set of his students
towards him/her is an instance of what kind of philosophy?
RATIONALIZATION: B. Progressivism. Progressivism is a philosophy of
learning by doing things. A. Realism C. Nationalism
B. Idealism D. Constructivism
11. Thea listened to the advice given by her sister to end the relationship
that she has with Gilbert. However, her sister learned that the advice she RATIONALIZATION: B. Idealism. Idealism is a philosophy which asserts that
has given was not followed and Thea decided to continue the relationship. what’s in the mind is the only reality. In this situation, the teacher inspires
This action of Thea is a manifestation of what kind of philosophy? his/her students through the values that he/she possessed. Values are
A. Essentialism C. Perennialism absolute are told to belong in realm of ideas. Hence, these are considered as
B. Existentialism D. Humanism ideas that exist only in the mind.

RATIONALIZATION: B. Existentialism. Existentialism is the philosophy of 16. John learns well through active interplay with others. His learning
subjectivity of selfhood increases when engaged in activities that has meaning in him. Which
and proclaims man’s freedom in the accomplishment of his destiny. philosophy can describe this?
A. Progressivism C. Realism
12. After finishing the degree in Education, Teacher M learns that learning B. Idealism D. Empiricism
never stops. In fact, she accumulates more knowledge after leaving the
portal of her alma mater. This typifies what kind of philosophy? RATIONALIZATION: A. Progressivism. Progressivism is a philosophy of
A. Constructivism C. Progressivism hands of learning or experiential learning. “Learning by doing”. Learning is
B. Perennialism D. Humanism based from the questions of one’s experience of the world. Hence, it is the
learner himself who thinks, solves, and gives meaning through his
RATIONALIZATION: C. Progressivism. Progressivism connotes change, individual experiences.
growth and development. It is an active form of philosophy for it continues
to evolve.
2
17. Your teacher is of the opinion that the world and everything in it are What kind of philosophy does this class uphold?
ever changing and so teaches you the skill to cope with the changes. Which A. Naturalism C. Progressivism
in his governing philosophy? B. Constructivism D. Reconstructivism
A. Experimentalism C. Idealism
B. Existentialism D. Realism RATIONALIZATION: D. Reconstructivism. Reconstructivism covers the
underlying factors that constitute reality or society. In this regard, students
RATIONALIZATION: A. Experimentalism. Experimentalism believes that are encourage to become involved in the problems whether political, social,
things are constantly changing. It is based on the view that reality is what or economical that confront the society and be able to arrive at solutions in
works right now and that goodness comes from group decisions. As a result, order to reconstruct society.
schools exist to discover and expand the society we live in. Students study
social experiences and solve problems. 21. A curriculum should only include those that have survived the test of
time and combine the symbols of literature, history, and mathematics. Thus,
18. Principal C shares this thought with his teachers: "Subject matter should curriculum like this contains values that are constant and universal. What
help students understand and appreciate themselves as unique individuals philosophy describes this kind of curriculum?
who accept complete responsibility for their thoughts, feelings, and A. Idealism C. Humanism
actions." From which philosophy is this thought B. Perennialism D. Essentialism
based? / Teacher V teaches her students to appreciate themselves as unique
individuals and are RATIONALIZATION: B. Perennialism. Perennialism maintains that
to accept the responsibility to their feelings, actions and thoughts. She education involves confronting the problems and questions that have
upholds the philosophy of challenged people over the centuries. Thus there is a need to study classical
. tradition of great books.
A. Perennialism C. Existentialism
22. Who among the following puts more emphasis on core requirements,
B. Essentialism D. Progressivism
longer school day, longer academic year and more challenging textbooks?
RATIONALIZATION: C. Existentialism. Existentialism is the philosophy of A. Perennialist C. Progressivist
subjectivity of selfhood B. Essentialist D. Existentialist
and proclaims man’s freedom in the accomplishment of his destiny.
RATIONALIZATION: B. Essentialist. Essentialism is an educational theory
19. Teacher H class is engage in problem solving activity which in a way is a rooted in classical realism and idealism which advocates curricular reform
reflection of the personal and social experiences. The purpose of this stressing the essentials of the basics like the 3R's.
activity is for the students to acquire the skills that can help them in solving
23. “Education is a continuous process of experiencing and of visiting or
their own real-life problems. What philosophy can best describe this?
reorganizing
A. Realism C. Progressivism
experiences “according to a Progressivist. What does it mean?
B. Idealism D. Existentialism
A. Education begins and ends in school.
RATIONALIZATION: C. Progressivism. Progressivism connotes growth and B. Education takes place anytime and anywhere. C. Education happens
development. In this manner learning comes best when things are being formally and informally.
experienced, that is learning by doing D. Education goes on throughout life.
things. It involves participation of the students and allows them to exercise
RATIONALIZATION: D. Education goes on throughout life. Progressivism
freedom.
connotes growth and development. It is an active form of philosophy for it
20. In the Social Science class of Teacher G, students identify the various continues to evolve.
social and economic problems that require urgent solutions. They not only
24. Mr. Cruz exemplifies an ideal teacher and son. He is competent in his
discuss the ways to address it but also agreed to participate in solving them.
field based on the various recognitions that he received from his school. As
3
a dutiful son, he assumes all the responsibilities of raising his family (this RATIONALIZATION: D. Existentialism. Conceives philosophy as something
includes his parents, siblings, and their own families) – giving all their that is felt by an individual because it’s concrete in itself or based on what is
needs and wants. The fulfilment of these is the neglect of the concrete. There is also personal involvement in this kind of philosophy.
responsibilities to himself. This attitude of Mr. Cruz is against what kind of
philosophy? 28. In line with the philosophy of Reconstructivism, which of the following
A. Existentialism C. Pragmatism should be given emphasis in teaching?
B. Constructivism D. Humanism A. To seek a better position in the society
B. To compare oneself with the less fortunate
RATIONALIZATION: D. Humanism. Humanism stresses personal culture, C. To become economically self-reliant
individual freedom, and development (the best way towards full and rich D. To designate one’s superiority over the others
lives).
RATIONALIZATION: C. To become economically self-reliant.
25. Teacher R ensures to put a certain amount of his monthly earnings in Reconstructivism aims to achieve social changes. This is a philosophy that
the bank. At the end of the school year, he used all his savings in visiting entails the identification of social problems, finding solutions for this and be
places rich of cultural heritages rather than of buying expensive clothes, a part of the solution.
jewellery, and latest gadgets. Favoring the former action over the latter
exhibits that Teacher R follows what kind of philosophy? 29. Joining organizations or clubs in school is requisite to granting of
A. Existentialism C. Behaviorism academic distinction under DECS Order No. 65, s. 1998. This shows school
B. Essentialism D. Progressivism community connection reflected in activities geared towards society's
needs. What philosophy is related to this? / Activities planned by
RATIONALIZATION: A. Existentialism. Existentialism is a philosophy that school clubs or organizations show school-community connection geared
gives a person the freedom to choose. It is a philosophy of subjectivity. towards society's needs. What philosophy is related to this?
A. Realism C. Existentialism
26. Teacher R ensures to put a certain amount of his monthly earnings in B. Progressivism D. Reconstructivism
the bank. At the end of the school year, he used all his savings in visiting
places rich of cultural heritages rather than of buying expensive clothes, RATIONALIZATION: D. Reconstructivism. Reconstructivism aims to achieve
jewellery, and latest gadgets. This shows that Teacher R is an advocate of social changes. This is a philosophy that entails the identification of social
what kind of philosophy? problems, finding solutions for this and be a part of the solution.
A. Pragmatism C. Naturalism
B. Perennialism D. Essentialism 30. Increase of time allotment in English, Math and Science in the
Elementary level under the New Elementary School Curriculum (NESC) as
RATIONALIZATION: B. Perennialism. Perennialism views education as a provided in DECS Order No. 1, s. 1993 contributes in the culturation of basic
recurring process based on eternal truths. Good education involves a search literacy by providing the needed essential skills. The related philosophy is .
for and understanding of truth which can all be found in the great works of A. Realism C. Idealism
civilization. B. Essentialism D. Perennialism

27. Teacher X rates the art work of her students not just on the merit of its RATIONALIZATION: B. Essentialism. Essentialism is an educational theory
appeal to the senses but also considers its uniqueness and the rooted in classical realism and idealism which advocates curricular reform
responsibility that every student has given in accomplishing the task. This stressing the essentials of the basics like the 3R's.
practice shows how Teacher X upholds this kind of philosophy:
A. Realism C. Idealism 31. Teacher A believes that the learner is the product of his environment.
B. Reconstructivism D. Existentialism Sometimes he has no choice. He/she is determined by his environment.
A. Rationalist C. Existentialist
B. Behaviorist D. Progressivist

4
RATIONALIZATION: B. Behaviorist. A behaviourist believes that human 1. As a parent and at the same time a teacher, which of the following will
behaviour can be best explained in terms of responses to external stimuli. you do to show your cooperation to a PTA project in your school to be
Also, education can be best achieved by modifying or changing student financed with the proceeds of the sales of the school canteen where food
behaviours in a socially acceptable manner through the arrangement of the prices are little bit higher?
conditions for learning. The control is obtained not by manipulating the a. Bring food for you and your children, but always make it a point to buy in
individual, but the environment. the school canteen. 💮 💮 ✔
b. Buy all you food in the school canteen but request for a discount
32. The curriculum is viewed as a means of developing desirable habits. It is
c. Bring food enough for you and your children but do not eat in the
recommended that the way /means to form these habits is through the
canteen
mastery of organized subject matter. Which philosophy is behind this
d. Buy all your food from the school canteen even if you cannot afford to do
educational view?
everyday.
A. Naturalism C. Realism
2. How can you help a habitual borrower of money get rid of his habit?
B. Idealism D. Pragmatism
a. Let him do something for you in return for the money you lent him
RATIONALIZATION: C. Realism. This educative process is viewed mainly as b. Direct him to others
the transmission of information and knowledge. The curriculum has the c. Do not lend him anymore 💮 💮 ✔
function of forming the body and this interrelationship gives rise to the d. Ask for a collateral for the cash he is loaning
dictum "sound mind in a sound body." 3. Periodic checks on student seatwork with a smile and pat on the shoulder
effectively reinforce good study habit is an example of __________
33. Teachers in school perform the role and responsibility of parents in the a. Discrimination reinforcement
development and education of the child. This stems from the belief that the b. Variable-ratio schedule
home is the primary agency in the education of the individual. Thus, c. Continuous reinforcement 💮 💮 ✔
teachers are regarded as surrogate parents along the loco parentis d. Fixed interval and variable-interval schedule
principle. Which philosophy espouses this view of education? 4. A person strives to work at a given task because of a need. Which of the
A. Pragmatism C. Idealism following situations can make a person strive to meet his needs?
B. Naturalism D. Realism
a. Minimize the unpleasant consequences of student involvement 💮 💮
RATIONALIZATION: B. Naturalism - the school is seen as the extension of ✔
home and teachers are deemed "surrogate parents". b. Utilize your own opinion as teacher in making final decisions in the
classroom
34. This philosophy believes that students need a passionate encounter c. Use unfamiliar materials as examples in order to initially arouse their
with the positive and negative phases of life like the joy and agony of love, curiosity
desirability of life, the inevitability of death, the anguish of freedom and the d. Ask pupils to submit test questions or reactions which you can select
consequences of choices and actions. Which philosophy advocates this view topics
in education? 5. The singing of the National Anthem in schools is an offshoot of the
A. Existentialism C. Essentialism philosophy of _________
B. Perennialism D. Realism a. Nationalism 💮 💮 ✔
c. Naturalism
RATIONALIZATION: A. Existentialism - it is a modern school of thought that
b. Pragmatism
grew out of the thoughts of European philosophers , particularly from Soren
d. Socialism
Kierkegaard, who believes that the central problem humanity is facing is
6. The environment in order to facilitate, learning must be interactive.
the ability to cope with its existence.
Which of the following best typifies this kind environment?
a. The child goes out and discovers for himself some rock or fossil 💮 💮

5
✔ 13. The Constitutional provision on language has the following aims
b. The child listens to a lecture on fossils given by the teacher EXCEPT:
c. The child summarize the section on fossils in his science textbook a. To make the regional dialect as auxilliary media of instructions in
d. The child copies a list of facts concerning fossils on the blackboard regional school
7. Social development means the acquisition of the ability to behave in b. To maintain English as a second language
accordance with __________ c. To make Filipino the sole medium of instruction 💮 💮 ✔
a. Social expectation 💮 💮 ✔ d. To make Filipino the national Language and medium of instruction and
c. social insight communication
b. Stereotyped behavior 14. The tendency to emphasize so much on school beautification to the
d. universal norms detriment of pupils performance illustrates the
8. When an adolescent combines ability to use deductive and inductive a. Filipino’s lack of seriousness
reasoning in the construction of realistic rules that he can respect and live b. Filipino’s lack of reflection
by, how does he perceive his environment? c. Filipino’s sense of humor
a. He views the world from his own perspective d. Filipino’s love for “porma” 💮 💮 ✔
b. He interprets events from a limited view 15. Which is NOT a characteristics of democratic discipline?
c. He views events apart from himself and other people 💮 💮 ✔ a. Child has opportunity to express his/her opinion
d. He views the world and himself through the eyes of other people b. Child given punishment is related to the misdeed
9. Who introduced the technique of using the drawing of a man as a c. Child understands the meaning of rules
measure of intelligence? d. Child obeys blindly 💮 💮 ✔
a. Aristotle 16. Who among the following stressed the processes of experience and
b. Herbart problem solving?
c. Good Enough 💮 💮 ✔ a. Dewey 💮 💮 ✔
d. Binet b. Aristotle
10. Which Republic Act provides government assistance to students and c. Hegel
teachers in private education? d. Plato
a. RA 7784 17. Which of the following reasons of measuring student achievement is
b. RA 6728 💮 💮 ✔ NOT valid?
c. RA 7836 a. To prepare feedback on the effectiveness of the learning process
d. RA 6675 b. To certify that students have attained a level of competence in a subject
11. The authoritarian setting in the Filipino home is reinforced by a area
classroom teacher who: c. To discourage students from cheating during test and getting high scores
a. Encourage pupils to ask questions 💮 💮 ✔
b. Prescribes what pupils should do 💮 💮 ✔ d. To motivate students to learn and master the materials they think will be
c. Is open to suggestions covered by the achievement test
d. Ask open ended questions 18. Which characterizes the perfectionist type of students?
12. Who among the following believes that learning requires disciplined a. Does not volunteer or initiate
attention, regular homework, and respect for legitimate authority? b. Give up easily
a. Essentialist 💮 💮 ✔ c. Rarely complete tasks
b. Perennialist d. Often anxious fearful or frustrated about quality of work 💮 💮 ✔
c. Progressivist 19. When a school decides to work on a thematic curriculum which should
d. Reconstructionist be out of the picture?

6
a. Peer collaboration b. Apply for teaching job where eligibility is not required to gain teaching
c. team teaching experience before taking the teachers board examination
b. Integration c. Prepare for the wedding she and her boyfriend have long planned to able
d. competition 💮 💮 ✔ to raise a family with children which they plan to rear as good citizen of our
20. Teacher wants to teach his pupils the technique on reading for country
information. Which technique should be used? d. Take the licensure examination for teacher and an oath to do her best to
a. Text structure help carry out the policies of the state 💮 💮 ✔
c. Story map 26. Parents are up in arms on the telephone bills that pay for sex calls. What
b. Prior knowledge is the solution to this problem?
d. SQ4R 💮 💮 ✔ a. The telephone company is to blame for this
21. In instructional planning, which among these three: unit plan, course b. The government restriction have no teeth
plan, lesson plan is (are) most specific? _________ plans. c. Parents allow this to make their children modern
a. Course and lesson d. Parents, school and students should discuss this openly 💮 💮 ✔
c. lesson 💮 💮 ✔ 27. Cooperative are encouraged is as many groups as possible. What agency
b. Course controls the different cooperatives?
d. unit a. Security and Exchange Commission 💮 💮 ✔
22. The use of drills in the classroom is rooted on Thorndike’s law of c. Commission on Audit
a. Readiness b. Department of Local Government
c. effect d. Bureau of Cooperative
b. Exercise 💮 💮 ✔ 28. Society and media know drinking starts off drug addiction. What should
d. belongingness be discussed in schools?
23. Positive interdependence as an element of collaborative learning means a. Drug addiction has been traced to drinking wine 💮 💮 ✔
that the students must: b. Nobody drinks at home except father
a. Learn to depend on each other to achieve a goal 💮 💮 ✔ c. TV ads show drinking is a source of fellowship
b. Depend on the diligent students d. High taxes on liquor will be deterrent to eventual drug use
c. Help one another in the individual test for everyone to pass 29. Cooperatives have branched out to consumer cooperatives. Schools
d. Be grouped heterogeneously have included the concepts of cooperatives. Where is it practiced?
24. Which of the following measures should a teacher do to a principal a. School book stores
whom she would like to file a case of sexual harassment without violating c. Schools uniform purchases
the relationship of the teacher and her superiors? b. School canteen 💮 💮 ✔
a. Present the case before competent authority and prepare to prove the d. Class stores
charge 💮 💮 ✔ 30. A student collapsed in her social studies class. It was found out that he
b. Write an anonymous letter to a higher school official to denounce the did not eat her lunch. What principle is shown in the situation
superior a. Psychological needs
c. Call a parent-teacher meeting and denounce the superior c. Somatotonic
d. Encourage the other teachers and students to hold a demonstration to b. Physiological need 💮 💮 ✔
oust the superior d. Safety need
25. Pick out the situation that illustrates the duty of a new teacher to the 31. The main function of a philosophy of education is to:
state: a. Aid the learner to build his own personal philosophy
a. Take a long vacation which she firmly believes she deserves after four b. Reconsider existing educational goals in the light of society’s needs
years of diligent study before taking the examination for teachers c. Provide the academic background prerequisite to learning

7
d. Define the goals and set the direction for which education is to strive 💮 38. Which is the least authentic mode of assessment?
💮 ✔ a. Paper-and-pencil test in vocabulary 💮 💮 ✔
b. Oral performance to assess student’s spoken communication ski8lls
32. Which technique (s) enable (s) a teacher to identify and eventually
c. Experiments in science to assess skill in the use of scientific methods
assists students with interpersonal difficulties?
d. Artist production for music or art subject
a. Anecdotal record
c. Cumulative record 39. In what period of a child is physical growth fastest?
b. Personal inventory a. Prenatal period
d. Sociogram** c. Early childhood **
b. Early adolescence
33. Teachers and students can participate in levels of computer use. Give
d. Prenatal and early adolescence
the order of computer use from simplest to complex?
a. Computer competency, computer literacy, competency expertise 40. How does fear affect the voluntariness of an act?
b. Computer literacy, computer expertise, computer competency a. Makes the act involuntary 💮 💮 ✔
c. Computer competency, computer expertise, computer literacy c. Increases voluntariness
d. Computer literacy, computer competency, computer expertise 💮 💮 b. No effect at all
✔ ✔ d. Lessens but not destroy voluntariness
34. Which one is considered the “Brain” of the microcomputer? 41. A group of people asserts that their culture is superior to another. This
exemplifies:
a. CPU 💮 💮 ✔
a. Cultural gap
b. Software
c. norm conflict
c. Video Screen
b. Cultural conflict
d. Keyboard
35. A group activity wherein one group representative presents the output d. Ethnocentrism 💮 💮 ✔
to the bigger group rather than individual pupils presenting the output is 42. A test consist of a graph showing the relationship between age and
known as: population. Follow a series of true-false items based on the graph. Which
a. Consensus decision type of test does this illustrate?
c. Jury trial a. Laboratory exercise
c. Performance
b. Composite report 💮 💮 ✔
b. Problem solving
d. Agenda
36. In the formulation of classroom regulations, which of the following d. interpretative 💮 💮 ✔
should a teacher refrain from doing? 43. Which curricular move served to strengthen spiritual and ethical
a. State classroom regulation as clearly as possible values?
b. Enlist student aid in the formation of classroom regulation a. Integration of creative thinking in all subject
c. Enforce classroom regulations consistently and fairly b. Reducing the number of subject areas into the skill subject
d. Teacher and the class should make as many regulations as possible 💮 c. Introduction of Value Education as a separate subject area 💮 💮 ✔
d. Re-introducing Science as all subject in Grade 1
💮 ✔
44. Which computer seems to have the most potential for the classroom?
37. Zero standard deviation means that:
a. Main frame computer
a. The students scores are the same 💮 💮 ✔
c. Microcomputer 💮 💮 ✔
b. 50% of the scores obtained is zero
b. Minicomputer
c. More than 50% of the score obtained is zero
d. LPC
d. Less than 50% of the scores obtained is zero
45. A teacher notices glaring wrong pronunciation of vowel sounds among
her students necessitating more practice. Which of the following activities
8
would be a most help? b. Perfect rights
a. Dictionary use d. Acquired rights
c. Assignments 52. In mastery learning the definition of an acceptable standard of
b. Review performance is called:
d. Drill 💮 💮 ✔ a. Alienable rights
46. When storage device is significantly more efficient in holding c. Inalienable rights
information b. Perfect rights
a. Hard disk 💮 💮 ✔ d. Acquired rights**
c. Floppy disk 53. A negative discrimination index means that:
b. Software
a. More from the lower group answered the test items correctly 💮 💮
d. Audio cassette

47. The Filipino tendency to resort to the easy way out from a term paper as b. The items could not discriminate between the lower and upper group
a course requirement by hiring a ghost writer or by passing a photocopied c. More from the upper group answered the test item correctly
term paper provide which Filipino traits? d. Less from the lower group got the test item correctly
a. Anticipation 54. Your teacher is of the opinion that the world and everything in it are
c. Pakikisama ever changing and so teaches you the skill to cope with the changes. Which
b. Ambivalence in his governing philosophy?
d. Lack of discipline 💮 💮 ✔ a. Experimentation
48. “ No pain, no gain.” This means that c. Realism
a. Only those willing to carry the crosses that life impose can share the joy b. Existentialism 💮 💮 ✔
of life 💮 💮 ✔ d. Idealism
b. One should be penitent every Friday by carrying his cross 55. For brainstorming to be effective which one should be out?
c. The more suffering in this life, the more one is assured of heaven a. Making use of the others ideas shared
d. One should look for suffering to save himself/ herself c.Non-threatening atmosphere
49. Which thrust on value formation is meant to help the students make use b. Teacher’s judge mental attitude 💮 💮 ✔
of their thinking and scientific investigation to decide on topics and d. Openness to idea
questions about values? 56. Which statement on spaced and massed learning is CORRECT?
a. Value inculcation a. Massed learning is better than spaced learning
c. Value clarification 💮 💮 ✔ b. Spaced learning is better than massed learning 💮 💮 ✔
b. Analysis c. Massed learning is as effective as spaced learning
d. Moral development d. Both massed learning and spaced learning are not effective
50. Which interactive teaching should be AVOIDED? 57. A teacher is a facilitator of learning and of the development of the youth.
a. Using “put down” strategy 💮 💮 ✔ Which practice is NOT in keeping with his role as facilitator?
c. Asking more divergent questions a. Considers the multiple intelligences of learners
b. Using multiple response strategy b. Humiliates misbehaving pupils 💮 💮 ✔
d. Asking more evaluative questions c. Dialogs with parents and with other members of the community
51. Rights which can not be renounced or transferred because they are d. Keeps himself abreast with educational trends
necessary for the fulfillment of man’s primordial obligations are called: 58. Which one indicates a teacher’s genuine enthusiasm and pride in
a. Alienable rights teaching?
c. Inalienable rights 💮 💮 ✔ a. Sticking to teaching for the moment that there are no better offers
b. Telling everyone that he went to teaching for there was no other choice
9
them c. The mean and median are equal
c. Engaging himself in continuing professional education 💮 💮 ✔ d. The scores are normally distributed
d. Belittling the re-numeration one gets from teaching 66. Which is implied by a negatively skewed score distribution/
59. In writing performance objective which word is NOT acceptable? a. The scores are evenly distributed from the left to the right
a. Manipulate b. Most pupils are underachieves
c. Delineate c. Most of the scores are high 💮 💮 ✔
b. Integrate d. Most of the scores are low
d. Comprehend 💮 💮 ✔ 67. A teacher discovers that a product of a certain bottling company brings
60. When is giving praise INEFFECTIVE? When IT? about damage to teeth. Much as he wants to share the products of his
a. Uses the accomplishment of peers as the context for describing a research, he could not because of harassment from all sides. Which
student’s present accomplishment 💮 💮 💮 ✔ teacher’s right is violated?
b. Provides information to student’s about their competence and the value a. Right to property
of their accomplishment c. Academic freedom 💮 💮 ✔
c. Focuses students attention on her own task relevant behavior b. Right to one’s honor
d. Shows spontaneity, variety and other signs of credibility d. Right to make a livelihood
61. Which statement applies when scores distribution is negatively skewed? 68. Why can the calculator do arithmetic? Because
a. The mode corresponds to a lower value a. A computer inside the calculator tells it how 💮 💮 ✔
b. The median is higher that the mode c. A typewriter inside does inside does it
c. The mode and median are equal b. A watch inside direct it
d. The mean corresponds to a high value 💮 💮 ✔ d. A TV inside shows it
62. The use of the process approach gives the student the opportunity to: 69. On which constitutional provision is the full or partial integration of
a. Learn on their own capable deaf and blind students in the classroom based? The provision on
c. Make use of laboratory apparatuses a. Providing citizenship and vocational training to adult citizen
b. Apply the scientific method 💮 💮 ✔ b. Protecting and promoting the rights of all citizen to quality education
d. Learn how to learn 💮 💮 ✔
63. A comprehension skill of higher level which may be inferred or implied c. Academic freedom
from reading is d. Creating scholarship for poor and deserving students
a. Picking out the main idea 70. Teaching in the cognitive, psychomotor and affective domains is based
c. Following direction on the concept that the learner is a:
b. Noting specific details a. Moral and feeling being
d. Drawing conclusion 💮 💮 ✔ c. Thinking, feeling and acting being 💮 💮 ✔
64. In the Preamble of the Code of Ethics of Professional Teachers, which is b. Maternal and an acting being
not mentioned about teachers? d. Spiritual and maternal being
a. Dully licensed professionals 71. Both Muslim and Christian value marriage but the Muslim practices
c. LET passers 💮 💮 ✔ polygamous marriage while the Christian practices monogamous marriage.
b. Posses dignity and reputation What is this called?
d. With high moral values a. Cultural relativism 💮 💮 ✔
65. What does a skewed score distribution mean? c. Ethical relativism
a. The scores are concentrated more at one end or the other end ** b. Acculturation
b. The mode, the mean and the median are equal d. Enculturation

10
72. Teacher wants to compare 2 concepts. With which technique can b. Symposium
accomplish this best? d. Consensus decision making 💮 💮 ✔
a. K-W-L technique 79. Which term applies to the search for related literature by computing
c. Spider web access of data bases of discs kept in libraries?
b. Venn diagram 💮 💮 ✔ a. On line research
d. Histogram c. Compact discs computer research 💮 💮 ✔
73. To build a sense of pride among Filipino youth which should be done? b. Manual research
a. Re-study our history and stress on our achievements as a people 💮 💮 d. Computer research
✔ 80. Which best indicates the effectiveness classroom activities?
b. Set aside the study of local history a. The laughter and enjoyment of students
c. Re-study our history from the perspective of our colonizers b. The application of concept learned in daily life 💮 💮 ✔
d. Replace the study of folklores and myths with technical subjects c. The utilization of varied techniques and approaches
74. When necessary conditions are present, the use of inductive method is d. The variety of instructional materials used
preferred because 81. The main purpose of the compulsory study of the Constitution in
a. There is greater active participation on the part of pupils 💮 💮 ✔ Philippine schools is to
b. It gives the teacher more time to rest a. Develop the students into responsible thinking citizens 💮 💮 ✔
c. It needs only few instruction materials b. Acquaint students with the historical development of the Philippine
d. Academic time is used wisely Constitution
75. Which is the best reason why teacher begins a lesson in Math by c. Prepare students for law making
checking and reviewing on the previous day’s assignment and provides d. Make constitutional experts of the students
practice and drills? 82. Some students who are high in the scholastic aptitude test have failed in
a. Check if parents guide their children in the making of assignment college. Some who are below the standards set for admission but who for
b. Make sure that the students understand the pre-requisite skills of the various reasons were admitted, attained satisfactory standings. This proves
lesson 💮 💮 ✔ that
c. Prepare the students for the mastery test a. Human beings are certainly predictable
d. Make learning interesting and enjoyable for students b. Admission tests are not accurate, hence should not be used
76. Which is a selective reading technique meant at getting at important c. Aptitude tests do not measure all factors important for success 💮 💮
facts very fast? ✔
a. Skim reading d. Aptitude test can be perfectly relied on
c. Oral reading 83. If the teachers pattern in questioning consists of calling on a student
b. Scanning 💮 💮 ✔ then asking the question
d. Silent reading a. All students may be encouraged to participate
77. For counseling to be successful which assumption must be AVOIDED? b. The student called to answer may be able to think well of his answer
a. The environment must provide assurance of confidentiality c. The rest of the class may just dictate the answer
b. The student is willing to participate in the process d. The rest of the class may not engage themselves in thinking of the answer
c. The counselor must be able to relate to the student 💮 💮 ✔
d. The counselor tells the student what to do 💮 💮 ✔ 84. In order to avoid disgrace, a pregnant, unmarried woman takes drug to
78. Which technique is most appropriate when a teacher wants a group to induce abortion. Is she morally justified to do that?
agree on a plan of action? a. Yes, it can save her and child from disgrace when he grows up
a. Composite report b. No, the unborn child can not be made to suffer the consequences of the
c. Agenda sins of his parents 💮 💮 ✔

11
c. No, the act of inducing abortion is bad in itself c. Mental retardation since he is culturally deprived
d. No, it is better to prevent the child from coming into the world who will d. Great change in IQ because he is culturally deprived
suffer very much due to the absence of a father 91. Which of the following is usually considered the most important factor
85. In which way does heredity affect the development of the learner? in a child’s observable classroom behavior ?
a. By placing limits beyond which the learner can not develop 💮 💮 ✔ a. Intelligence
b. By compensating for what environment fails to develop c. Self concept 💮 💮 ✔
c. By blocking the influence of environment b. Heredity
d. By providing equal potential to all d. Cultural background
86. The cultivation of reflective and meditative skills in teaching is an 92. Section 5, article XIV, of the Constitution states that academic freedom
influence of shall be enjoyed in
a. Taoism a. Public assemblies
c. Confucianism c. All levels of learning
b. Shintoism b. State colleges and universities
d. Zen Buddhism 💮 💮 ✔ d. All institution of higher learning 💮 💮 ✔
87. A child refuse to obey orders or displays negativism as a development 93. A teacher who subscribes to the pragmatic philosophy of education
trait. How may you best handle him? believes that experience should follow learning in her teaching, she
a. Take every opportunity to praise him for every positive attitude therefore exerts effort in
displayed 💮 💮 ✔ a. Encouraging learners to memorize factual knowledge
b. Detain him after office hours for him do to what he has been ordered to b. Equipping learners with the basic abilities and skills
do c. Requiring learners full mastery of the lesson
c. Insist on compliance to the same degree required of pupils d. Providing learners opportunities to apply theories and principles 💮 💮
d. Avoid giving him orders if you do and he objects take back the order ✔
88. Which term refers to the collection of students products and 94. Freud expounded that there is a period when young boys experience
accomplishment for a period of evaluation purposes? rivalry with their father for their mother’s affection. This is
a. Portfolio 💮 💮 ✔ a. Oedipus complex 💮 💮 ✔
c. Anecdotal record c. Achilles syndrome
b. Observation report b. Electra complex
d. Diary d. Cassandra syndrome
89. For comparing and contrasting which graphic organizers is most 95. Education is a life long process. This simply means that education
appropriate? a. May take place formally or informally to enable the individual to grow
a. Cycle b. May take place anywhere and any time the individual so desires
c. Story map c. Is a continuous process of experiencing and reorganizing experiences
b. Web
💮 💮 ✔
d. Venn Diagram 💮 💮 ✔ d. Take place in the school where the individual is exposed, self contained
90. If a resilient child with superior intelligence is reared in a poor experiences
environment the probable outcome would be 96. The tendency to imitate elders is very strong in the early childhood
a. No change in IQ because environment deprivation has nothing has stage. Teachers should therefore be very good
nothing to do with intelligence a. Counselors
b. Slight change in IQ although he can overcome frustration and obstacle c. Disciplinarians
💮 💮 ✔ b. Role models 💮 💮 ✔
d. Facilitators of learning
12
97. How is Values Education offered in the National Secondary Education specifically that of being able to solve the problem he faces individually or
Curriculum? collectively
a. Emphasized in Science and Technology a. Disciplinarianism c. Experimentation 💮 💮 ✔
b. As a separate subject 💮 💮 ✔ b. Developmentalism d. Rationalism
c. Integrated in all subject areas 104. Which of the following abilities is stressed by humanistic education?
d. Integrated with Technology and Home Economics a. Learn the different philosophies of education
98. The NSEC orients secondary education to b. Develop man into a thinking individual
a. The teaching of the national symbols c. Enjoy the great works of man such as the classics
b. The development of competencies and values for social living 💮 💮 d. Make man distinctly civilized educated and refined 💮 💮 ✔
✔ 105. An appreciation lesson is one that is designed to lead the class to
c. Health values development conduct and enjoy something. Which of the following statements closely
d. National development requirement and reflects research based direction approximate the meaning of the above?
99. The child cannot distinguish abstracts during the sensory motor of a. An appreciation lesson should be a lesson in values
development. Which of these techniques should a teacher apply to b. Appreciation lessons help pupils weigh and clarify values
accommodate learning? c. One cannot fully appreciate what one does not understand or enjoy
a. Make use of individualized instruction d. A teacher should plan lessons that will guide children to appreciate what
b. Explain the lesson very well is beautiful 💮 💮 ✔
c. Utilize concrete objects to clarify concept 106. Which of the following is the best time for a teacher to set up routine
d. Provide variety of educational toys 💮 💮 ✔ activities that will contribute to effective classroom management?
100. Which of these systems of learning includes ways and methods which a. As soon as the students have established
are used in preserving and building certain within cultural communities? c. During his homeroom days
a. Non-formal learning b. Daily at the start of the session
c. Cultural learning ** d. On the very first day of school 💮 💮 ✔
b. Multi-level learning 107. In large classes where little of the work pupils can be individualized,
d. Indigenous learning the most effective and practical ways to individualize instruction is to
101. Which of the following statements is TRUE in the use of experiments a. Devise group activities which afford every pupil an opportunity to work
and demonstrations in teaching science at his own 💮 💮 ✔
a. It is valuable if used in the context of a lesson that related observation to b. Give the pupils freedom to launch individual projects
other information 💮 💮 ✔ c. Assign homework and check it regularly
b. It should be encouraged in elementary school since the concept the d. Assign program material for out-of-class hours
encompass are difficult for your children 108. Which of these is the MOST important principle that a teacher should
c. It is as valuable as teaching by lecturing follow in initiating program for positive reinforcement?
d. It is less valuable than teaching through inquiry and discussion a. Make sure the reward comes immediately after the appropriate behavior
102. Identical twins are more alike than fraternal twins. Which of the 💮 💮 ✔
following statements/ principle is supported by this? b. Punish negative behavior and reward positive behavior
a. Environment affects both fraternal and identical twins c. Provide regular opportunity for socially acceptable behavior
b. Intelligence hinges in physical structure d. Consider peer approval and recognition
c. Heredity has a part in determining intelligence 💮 💮 ✔ 109. The trend of focusing attention on the child’s interests, abilities and
d. Intelligence is determined partly by pre-natal nutrition needs and on the improvement of community living necessitate the use of
103. Which of theses philosophers is reflective of that of Dewey’s which the
stresses the development of an individual capable of reflective thinking a. Discovery approach

13
c. Integrative approach 115. The educational implementation of research findings relative to the
b. Conceptual technique ability of dull learners and bright learners to organize and generalize is for
d. Project method 💮 💮 ✔ teacher
110. The best way the teacher can see the appropriateness of an a. To make the bright learners guide the dull ones in learning generalize
instructional materials is to b. To make the bright learners to generalize and the dull ones to memorize
a. Consider its technical quality c. To give the dull learners more concrete experiences to serve as basis for
c. Consider its availability generalizing
b. Try it out before using it in class 💮 💮 ✔ d. To give both the dull and bright learners concrete and abstract
d. Consider its cost experiences to serve as basis for generalizing 💮 💮 ✔
111. Tasks analysis involves the breaking down of a learning task into 116. Which of the following will you do the FIRST to establish good class
subtasks or sub skills. Given a task to retell a story, which of the following management?
skills is NOT needed? a. Discuss the required rules for proper class behavior 💮 💮 ✔
a. To disseminate information 💮 💮 ✔ b. Discuss the work plan for the year
c. To identify topic sentences c. Prepare a seat plan
b. To outline a selection d. Train the class in the distribution of material
d. To arrange events in sequence 117. A student was diagnosed to have a high IQ but is falling in his academic
112. You are assigned to teach students with varied abilities. You want to subject. What should the teacher do to help him?
teach a more homogenous grouping. Which type of grouping will tend to a. Talk to his parents
benefit your students? b. Examine his study habits
a. Mixed ability grouping c. Talk the student and find out his problem 💮 💮 ✔
c. With-in class ability grouping 💮 💮 ✔ d. Refer him to the guidance counselor
b. Low ability group 118. Which is the TRUE foundation of the social order?
d. High ability grouping a. Strong, political leadership
113. Which of the following examples illustrate the use of questions to focus b. The reciprocation of rights and duties 💮 💮 ✔
pupil attention on the key points of the lesson? c. Equitable distribution of wealth
a. Why are machine made goods cheaper than those made by hand? 💮 💮 d. Obedient citizenry
✔ 119. When do test, inventories and career information become effective for
b. What is Rizal Park known for? counseling services?
c. Have you ever enjoyed watching the clouds on a bright day? a. When the data generated are interpreted on time by professionally
d. Who came while I was writing on the blackboard? competent person 💮 💮 ✔
114. The new teacher entered a noisy classroom. She shouted immediately b. The psychological test result are still valid and reliable
at the students desperately trying to get order and discipline. Since then the c. When the records are updated
teacher has not controlled the class. Which is the most probable cause of the d. When the records are kept for ready reference when needed
teacher’s failure? 120. Which of the following is a major advantage in using arithmetic mean?
a. The students reaction to the teacher is the consequence of her behavior a. It is more commonly used than other measures
b. Rules are not defined and procedures to sustain order is not put into b. It discriminates between the lowest and the highest
place c. It is simple to compute 💮 💮 ✔
c. The new teacher wants to show the class who is authority d. It is more than stable than the median
d. The class wants to test the ability and patience of the teacher. 💮 💮 121. In preparing a multiple choice test how many options would be ideal?
✔ a. It is more commonly used than other measures
b. It discriminates between lowest and the highest
14
c. It is simple to compute c. 6 to 9 years
d. It is more than stable than the median d. 3 to 6 years
122. Learners often find it much easier to fit into a new social situation 128. Which of the following is the best situation wherein you can balance
when given encouragement and support. How can this be done? responsibility and accountability?
a. By giving him room responsibility a. A teacher paid on an hour basis, takes her time with the subject matter till
end of period
b. By assigning “peers or Buddies” to him 💮 💮 ✔
b. A teacher paid on an hour basis, teaches as much as she could for
c. By giving him special help
d. By discovering his new interest duration of the period 💮 💮 ✔
123. Who expounded on the need to study the child carefully for c. A teacher paid on an hour basis, spends most of the time on the latest
individualized instruction? gossips in showbiz
d. A teacher paid on an hour basis, entertain her students with stories till
a. Da Feltre 💮 💮 ✔
the end of the period
b. Erasmus
129. You have a pupil who is so talkative, naughty and aggressive that he is
c. Boccacio
a burden to the entire members of the class. How would you remedy this
d. Ascham
problem?
124. Which of the following should a teacher do if she cannot pay the
a. Call the parents for dialogue
monthly installment of an appliance she got from a department store in
c. Reprimand him always
their town?
b. Report the case to the principal
a. Reject any notice of demand for payment to make the impression that she
did not receive d. Talk to him seriously 💮 💮 ✔
b. Move to another neighborhood to escape payment 130. What should a teacher do before constructing items for a particular
c. Inform the manager of the store personally and make a satisfactory test?
arrangement of payment on or before the due date of payment 💮 💮 a. Prepare a table specifications 💮 💮 ✔
✔ b. Review the previous lessons
d. Offer the return the used appliance to the store on the condition that she c. Determine the length of time for answering it
will be refunded on the monthly installment she paid d. Announce to students the scope of the test
125. Which of the following will you recommended to a senior high school 131. Under which of the multiple choice type of test can this question be
scholar who is impregnated by a fellow student? classified? ‘ Which of the following statements expresses this concept in
a. Tell her parent about her condition 💮 💮 ✔ different forms?
b. Stop schooling till after she gives birth a. Association 💮 💮 ✔
c. Direct her to an abortion clinic b. Definition
d. Force her boyfriend to marry her c. Difference
126. The government prescribes a higher percentage on the administration d. Cause
of educational institution to Filipino citizens in order to 132. Of the following types of test which is the most subjective in scoring?
a. Minimize the unemployment problem a. Matching type
b. Procedure globally competitive graduates 💮 💮 ✔ b. Simple recall
c. Protect the rights of the citizen c. Multiple choice
d. Ensure the teaching of Filipino d. Essay 💮 💮 ✔
127. Teacher should bear in mind that the period of greatest mental 133. In which of these research methods can the researcher control certain
development is from: variable?
a. 9 to 12 years a. Experimental 💮 💮 ✔
b. 12 to 15 years 💮 💮 ✔ b. Ex post facto

15
c. Descriptive b. Standard deviation 💮 💮 ✔
d. Historical d. Distribution of raw scores
134. During the first grading period, a student obtained failing marks in five 141. The theory of identical elements in learning holds that transfer is
academic subjects. Which of the following tests would best explain his facilitated when the ______
performance? a. Teacher uses different teaching devices
a. Mental ability b. Learner has a memory of specific responses
b. Personality c. Development task is easily identifies
c. Attitude d. Experience is similar to the application situation 💮 💮 ✔
d. Aptitude 💮 💮 ✔ 142. If this need is not met the adolescent tends to be critical and always
135. Measuring the work done by gravitational force is a learning task. At tries to find fault. This is the need
what level of cognition ? a. For recognition 💮 💮 ✔
a. Application 💮 💮 ✔ c. to belong
b. Knowledge b. For adventure
c. Evaluation d. for material security
d. Comprehension 143. The way a child talks, walks of manifest, gestures may have been
136. Setting up criteria for scoring test is meant to increase their ________ learned from models he had been exposed. This explains
a. Objectively ** a. Affective
b. Reliability b. Insight
c. Validity c. Social 💮 💮 ✔
d. Usability d. Cognitive
137. Which of the following you will do to an examinee you caught cheating 144. Audio-visual aids are used in classroom teaching to __________
and who offered to a certain sum of money to keep quiet a. Help make learning more permanent
a. Motion him to keep quiet and watch for him after the examination c. Help clarify important concept
b. Confiscate his test paper and report him to the examination supervisor b. All of these 💮 💮 ✔
d. Arouse and sustain student’s interest
💮 💮 ✔
145. Which of the following is the MOST important purpose for using
c. Announce to all examinees the name of the cheater
achievement test? To measure the _________
d. Ignore him but let him feel you saw him
138. Which of the following is the best situation wherein you can balance a. Quality and quantity of previous learning 💮 💮 ✔
rights and authority? c. Educational and vocational aptitude
a. Allow all their only daughter’s suitor to come and go as she pleases b. Quality and quantity of previous teaching
b. Censor all their only daughter’s suitor d. Capacity for future learning
c. Choose a life-partner for their only daughter 146. Which of the different types of test covers a wide variety of objectives?
a. True-false
d. Caution their only daughter’s choice of a boyfriend 💮 💮 ✔
c. Matching
139. In testing which of the following is referred to as cultural bias?
b. Multiple choice 💮 💮 ✔
a. Test items are more familiar with some culture 💮 💮 ✔
d. Essay
b. Some culture do better on test than others
147. In a multiple choice test, keeping the options brief indicates ____________
c. Test will show who is more cultured
a. Inclusion in the item irrelevant clues such as the use in the correct
d. Cultured people do better on tests
answer
140. Which is the most obvious and familiar way of reporting variability?
b. Non inclusion of option that mean the same
a. Range between highest with some culture
c. Standard error of the mean c. Plausibility and attractiveness of the item 💮 ✔
16
d. Inclusion in the item any word that must otherwise repeated in each This was a project initiated by
response a. Arsenio Lacson
148. Which of these criteria is the most important in test constructions? b. German Moreno
a. The stem should contain the central problem c. Antonio Villegas
b. Items should be congruent with the objectives 💮 💮 ✔ d. Ramon Bagatsing
c. A table of specification should be prepared Answer: C
d. Options should be of almost the same length 6. Which among the sounds below is voiceless?
a. /b/
b. /z/
1. It is a collection of religious poetry written by Rabindranath Tagore. c. /g/
a. Mahabharata d. /p/
b. Gitanjali Answer: D
c. The Ramayana 7. “Only the heart can see rightly.” This statement is lifted from
d. Bhagavad Gita what particular novel?
Answer: B a. The Prince and the Pauper
2. Robert Frost wrote the poem Acquainted with the Night from b. The Little Prince
which the stanza is taken: c. The Right One
I have been one acquainted with the night. d. The Pearl
I have walked out in rain-back in rain. Answer: B
I have out walked the farthest city light 8. He was the American President who said, “Ask not what America
The poet in the stanza talks of? will do for you, but what together we can do for the freedom of
a. isolation and loneliness man.”
b. joy getting out of the house a. Gerald Ford
c. youthful delight playing in the rain b. Franklin Roosevelt
d. happiness in having been acquainted with the night c. Henry Truman
Answer: A d. John F. Kennedy
3. Which is the BEST WAY to write the underlined portion of this Answer: D
sentence? Researchers also speculate that some teachers might 9. History is the witness that ______ passing of time.
have given boys more computer time because parents and teachers a. testifies
expected boys to need computers for future careers. b. will testify
a. expected c. testifies for
b. expecting d. testifies for the
c. will expect Answer: D
d. will have expected 10. When I met Liza yesterday, it was the first time I _____ her since
Answer: D Christmas.
4. The wounded soldiers were visited by the president who a. saw
honoured them with ____ for their _____. b. had seen
a. medals – valor c. have seen
b. gun salute – bravery d. have been seen
c. appointments – dedication Answer: D
d. money – sacrifice 11. The commander ordered his men to hold on the fort. What was
Answer: A the message?
5. Every June, Manila has its festival of outstanding Filipino films.
17
a. Surrender in arms b. Triumph
b. Keep on with the fight c. Indecision
c. Disregard the peace negotiation d. Aggression
d. Rescue the hostage victims fast and early Answer: A
Answer: B 18. I suggest that he _____ in the room for one week.
12. “She is a vision of feminine pulchritude.” This stands for the a. Stay
following EXCEPT b. Stayed
a. Loveliness c. Staying
b. Comeliness d. Stays
c. Homeliness Answer: A
d. Physical beauty 19. The manager told his workers, “We have to reduce our
Answer: C workforce.” What did he mean?
13. The Nibelungenlied is a Workers are free to leave
a. Latin Myth Workers are warned of possible lay off
b. Chinese legend Workers have to double time on their jobs
c. Russian folk song Workers should submit themselves to a reducing gym
d. Medieval German epic Answer: B
Answer: D 20. The parent remarked, “__________ I come late, just lock the door.”
14. An association wherein the name of something is substituted by a. In the absence
something that represents it. b. In the process
a. Metonymy c. In the event
b. Comparison d. In the case
c. Euphemism Answer: C
d. Personification 21. The copyreader found the news story boring. He found it full of
Answer: A _____.
15. Because the moon rotates on its axis at the same time as it ______ a. Adjectives
around the earth, we see the same side b. Verbs
a. Revolve c. Pronouns
b. Revolves d. Adverbs
c. Is revolving Answer: C
d. Has been revolving 22. There were three guests on the stage. They were made up of a
Answer: B parent, the governor and the principal. Who should be
16. In English verse, a poetic foot having 1 stressed syllable acknowledged first by the valedictorian?
followed by 1 unstressed syllable is ______. a. The classmates
a. Trochaic b. The principal
b. Iambic c. The governor
c. Dactylic d. The parent
d. Anapaestic Answer: C
Answer: A 23. What is suggested in the opening line? June 13, 1986 - they
17. Senators were accused by activists of washing their hands with came from all over America - 200,000 heroes strong, with their
the perfumes of Arabia. This state is commonly known as families.
a. Guilt a. The writer holds great admiration for the veterans

18
b. The writer is opposed to the Vietnam War Drum on your drums, batter on your banjos, sob on the long cool
c. The writer was a veteran of the war winding saxophones. Go to it, O jazzmen.
d. The writer is a flag-waving patriot Which words illustrate alliteration?
Answer: A a. Batter and banjos
24. A readability mismatch happens when the reading levels of b. Sob and winding
books exceed the reading levels of the students. In this situation, the c. Long and cool
students experience frustration and they fall short of the expected d. To and it
or desired output. A student who finds himself/ herself in such a Answer: A
mismatch will likely do which of the following? 29. What is meant by AT SIXES AND SEVENS in this sentence?
a. Give an intelligent critique of the selection or story read We moved into the house last week, but I'm afraid everything, is
b. Write a comprehensive reaction paper regarding the selection or still at sixes and sevens.
story read a. The things have not been shipped.
c. Present an argument that the selection or story read was not b. In a state of confusion
properly written c. In an orderly manner
d. Manifest an expected and commensurate emotional reaction to d. The boxes are still intact
the selection or story read Answer: B
Answer: C 30. What is the mood of these lines?
25. What does this mean: Excuses are for losers… those who take Daylight, I must wait for the sunrise
responsibility for their actions are real winners in life. This tells of? I must think of a new life
a. losers often fail because they find reasons for losing And I mustn't give in.
b. a winner can also be a quitter When the dawn comes
c. accountability of one’s action tells of bravery tonight will be a memory, too
d. excuses are needed to justify any failure And a new day will begin.
Answer: C a. Afraid
26. A couple accepted a wedding invitation. They showed pleasure b. Sarcastic
in these remarks c. Depressed
a. All guests congratulated the organizers and the couple d. Hopeful
b. Guests came in and out of the ceremonies Answer: D
c. The ceremonies were very impressive Below is the LET Reviewer for General Education GENED: Filipino
d. The couple felt uneasy with the priest Part 1. We encourage readers/ reviewees to use the comment boxes
Answer: C after the article for discussion.
27. What correction should be made to this sentence? 1. Sinabi ni Carlos P. Romulo sa isa niyang akda, “Ang Pilipino ay
One of the theories is that the first child receives more of the may dugong maharlika.” Ano ang kahulugan nito?
parents' attention than other children so first-borns tend to be a. Ang Pilipino ay nanggaling sa malayang lahi
more intellectual. b. Ang Pilipino ay sadyang mabuti ang budhi.
a. Change is to are c. Ang Pilipino ay galing sa mayamang lahi
b. Insert a comma after children d. Ang Pilipino ay madaling maipagbili
c. Change parents' to parent's Answer: A
d. Change theories to theory's 2. Alin sa mga sumusunod ang pinakatamang pangungusap?
Answer: B a. Ang mga kabataan sa lansangan ay naglalaro at nagtatakbuhan sa
28. Carl Sundburg wrote "Jazz Fantasia" which has for its first lansangan.
stanza: b. Ang kabataan ay naglalaro kung maliwanag ang buwan at

19
nagtatakbuhan sa lansangan. Answer: D
c. Ang mga kabataan ay naglalaro at nagtatakbuhan sa lansangan 8. Kabaliwan at paglulustay ang inyong ginagawa taon-taon. Higit
kung maliwanag ang gabi. na marami ang maralitang nangangailangan ng salapi at dunong.
d. Ang mga kabataan kung maliwanag ang buwan ay nagtatakbuhan Ang nagsasalita ay
sa lansangan at naglalaro. a. Kuripot
Answer: C b. Matipid
3. Piliin ang gawi ng pagsasalita: Kasiyahan ko nang makitang c. Maramot
kayo’y nagmamahalan. d. Praktikal
a. Pangarap Answer: D
b. Pagkontrol ng kilos 9. Nasa anong kaganapan ng pandiwa ang pangungusap?
c. Pagkuha ng impormasyon Naglaro ng basketball sa Rizal Stadium ang koponan ng aming
d. Pagbabahagi ng damdamin pamantasan.
Answer: D a. Sanhi
4. Ang kaugnayan ng pagkakapatay kina Burgos, Gomez at Zamora b. Tagaganap
sa panitikang Pilipino ay c. Kagamitan
a. Nanatiling masigla ang diwang Pilipino d. Ganapan
b. Nakagising sa damdaming makabayan ng mga Pilipino Answer: D
c. Natutong lumabag sa batas at lumaban sa may kapangyarihan ang 10, Sa aling salita magkakaroon ng saglit na paghinto kung
mga Pilipino pinagpipilitang si Rose ang nakabasag ng pinggan?
d. Naimpluwensyahan ang diwang alipin ng mga Pilipino Hindi si Rose ang nakabasag ng pinggan.
Answer: B a. Rose
5. Laging UMUUKILKIL sa isipan ng ama ang nasirang pangako ng b. Hindi
anak. c. Nakabasag
a. Sumasagi d. Pinggan
b. Gumugulo Answer: B
c. Bumubuhay 11. Anong tayutay ang tinutukoy sa pahayag.
d. Sumasapi Durog ang katawang bumagsak sa semento si Miguel.
Answer: B a. Pagtutulad
6. Ang Kagawaran ng Ugnayang Panlabas ng Pilipinas ay gulong- b. Pagbibigay katauhan
gulo kapag may giyera sa ibang bansa. Ano ang unang c. Pagmamalabis
ginagampanan ng ambassador ng bansa? d. Pagwawangis
a. Bilangin ang mga nasugatan at nasawi Answer: C
b. Alamin ang mga tirahan ng mga Pilipino sa bayang iyon. 12. Sino ang pinagkalooban ng karangalan bilang “Unang Tunay na
c. Ipunin ang mga maykaya at ipalipad pauwi. Makata” noong 1708?
d. Bayaang magsipag-uwian sa sariling sikap ang bawat isa a. Jose dela Cruz
Answer: B b. Felipe de Jesus
7. Ano ang pokus ng pandiwa sa pahayag na “Bumili ng bagong c. Francisco Balagtas
sasakyan si Angelo”? d. Jose Corazon de Jesus
a. Pokus sa direksyon Answer: B
b. Pokus sa kagamitan 13. “Magtatrabaho ako at ikaw ay mag-aaral upang makatapos ka ng
c. Pokus sa sanhi pag-aaral.” Anong uri ng pangungusap ito?
d. Pokus sa aktor a. Payak

20
b. Tambalan d. Malayo-Polinesyo
c. Hugnayan Answer: D
d. Langkapan 20. Ano ang katumbas ng “Dekalogo” ni Apolinario Mabini na
Answer: D nagsasaad ng aral sa Filipino?
14. Mag-aalas-singko na _____ umaga _____ magising siya. a. Mosaic Law
a. ng – ng d. Code of Ethics ni Kalantiaw
b. nang – nang c. New Society ni Pres. Marcos
c. ng – nang d. Code of Citizenship ni Pres. Quezon
d. nang – kapag Answer: A
Answer: C 21. Siya ay hinirang na taga-sensus ng bahay-bahay. Ano ang
15. Ang butong tinangay ng aso, walang pagsalang nalawayan ito. kanyang nalikom?
Ang kaisipang ito ay tumutukoy sa katotohanan ng ______. a. Ang bilang ng tao sa bahay
a. Pagnanakaw b. Ang kayamanan ng may-bahay
b. Pagtatanan c. Ang datos tungkol sa mga bata sa bawat bahay
c. Pagpapakasal d. Ang datos tungkol sa mga naninirahan sa bawat bahay
d. Pakikipagkaibigan Answer: D
Answer: B 22. Kami ang kabataang siyang magiging pag-asa ng bayan. Paano
16. Anong uri ng pagbigkas ang salitang “dambuhala”? ginamit ang salitang may salungguhit?
a. Malumi a. Pagtukoy
b. Mabilis b. Pagpuri
c. Maragsa c. Panghalip
d. Malumay d. Pagmamalaki
Answer: A Answer: A
17. Ang katawagan sa pangngalan, pang-abay, pang-uri at pandiwa 23. Walang tubig kahapon. Ito ay pangungusap na:
ay? a. May paksa
a. Palabuuan b. Walang pandiwa
b. Pangkayarian c. May panaguri
c. Pangnilalaman d. Walang paksa
d. Palaugnayan Answer: D
Answer: C 24. Ipinagmamalaki mo siya, BAHAG naman pala ang kanyang
18. Ang panukalang inihain niya ay lubhang malalim at mahirap BUNTOT. Ang ibig sabihin ng salitang may malaking titik ay:
arukin. a. Kuripot
a. Abutin b. Traydor
b. Unawain c. Duwag
c. Sukatin d. Mahiyain
d. Tanggalin Answer: C
Answer: B 25. Sabihin ang gawi ng pananalitang ito: “Bawal tumawid, may
19. Ang wikang Filipino ay hawig sa mga wika sa Asya. Alin dito ang namatay na dito!”
pinagmulan ng wikang Filipino? a. Pananakot
a. Bahasa b. Pagtukoy
c. Nihonggo c. Babala
d. Mandarin d. Paalala

21
Answer: C north at the rate of 30 kph and the other travels east at 40 kph. How
26. Pinakamahalagang nobelang Pilipino sa maraming taon na many kilometers apart are the buses at 10 pm?
nalimbag noong 1906 at tumalakay nang masinsinan sa paksang a. 140 km
puhunan laban paggawa at sa sosyalismo ang _____. b. 100 km
a. Luha ng Buwaya c. 70 km
b. Banaag at Sikat d. 50 km
c. Ibong Mandaragit Answer: B
d. Pangginggera 2. Calculate the mean absolute deviation of the following numbers:
Answer: B 60, 80, 100, 75 and 95
27. Ayon kay Balagtas, “ang laki sa layaw, karaniwa’y hubad” kaya a. 12.4
ang mga bata ay b. 14.2
a. jeproks c. 16.1
b. nag-aartista d. 18.9
c. nakapagtatapos sa pag-aaral Answer: A
d. hindi sumusunod sa magulang 3. Which of the following is the factorization of the binomial x2 -
Answer: D 42?
28. Noong taong 1962, ano ang pagbabago sa paglimbag ng diploma a. (x + 4)(x + 2)
at sertipiko ng pagtatapos? b. (x – 4)2
a. Pinahihintutan ang pribadong paaralan na maglimbag sa wikang c. x(x + 2x + 2)
Ingles d. (x – 4)(x + 4)
b. Nilimbag sa Tagalog ang diploma sa di-Tagalog na bayan Answer: D
c. Nilimbag sa Filipino ang diploma ngunit may Ingles 4. What value of x will satisfy the equation: 0.4(5x - 1470) = x?
d. Nalimbag sa Filipino ang diploma a. 490
Answer: D b. 2,130
29. Ang gintong panahon ng mga manunulat noong panahon ng c. 1470
Amerikano ay batid sa uring Answer: D
a. Sanaysay 5. Which of the following has the greatest value:
b. Nobela a. 3 + 32 + (3 + 3)2
c. Panulaan b. 33
d. Maikling kwento c. [(3 + 3)2]2
Answer: D d. (3 + 3 + 3)2
30. Alin sa mga sumusunod ang may wastong gamit ng tinig ng Answer: C
pandiwa? 6. The average of 5 different counting numbers is 20. What is the
a. Ang hinog na papaya na kinuha sa puno ni Marie. highest possible value that one of the numbers can have?
b. Kinuha ni Marie ang hinog na papaya sa puno. a. 20
c. Kinuha sa puno ang hinog na papaya ni Marie. b. 40
d. Papayang hinog ang kinuha sa puno ni Marie. c. 30
Answer: B d. 90
Below is the LET Reviewer for General Education GENED: Answer: D
Mathematics Part 1. We encourage readers/ reviewees to use the 7. Three brothers inherited a cash amount of P62,000 and they
comment boxes after the article for discussion. divided it among themselves in the ratio of 5:4:1. How much more
1. Two buses leave the same station at 8:00 pm. One bus travels is the largest share than the smallest share?

22
a. P75,000 b. 2 and 1/3 hours
b. P30,000 c. 3 hours
c. P24,800 d. 7 and 1/2 hours
Answer: C Answer: D
8. What is the missing terms in the series 5, 20, 80, ___,1280, ___, 20, 14. How much greater is the sum of the first 50 counting numbers
480? greater than the sum of the first 100 counting numbers?
a. 50;210 a. 110
b. 40;160 b. 3,775
c. 35;135 c. 3,155
d. 320;5120 d. 1200
Answer: D Answer: N
9. At what rate per annum should P2400 be invested so that it will 15. Which of the following has the largest value?
earn an interest of P800 in 8 years? a. 85
a. 6 ½ % b. 39
b. 5 ½ % c. 65
c. 4.17 % d. 94
d. 6 % Answer: A
Answer: C 16. A water tank contains 18 liters when it is 20% full. How many
10. The area of a rectangle is (x2 + 2x - 8). If its length is x + 4, what liters does it contain when 50% full?
is its width? a. 60
a. x + 2 b. 30
b. x - 2 c. 58
c. x + 1 d. 45
d. x + 6 Answer: D
Answer: B 17. The edges of a rectangular solid have these measures: 1.5 feet
11. What is the value of 12⅙ - 3 ⅜ - 5 ⅔ + 20 ¾? by 1½ feet by 3 inches. What is its volume in cubic inches?
a. 21 1/8 a. 324
b. 22 b. 225
c. 23 7/8 c. 972
d. 21 d. 27
Answer: C Answer: C
12. The vertex angle of an isosceles triangle is 20°. What is the 18. In a certain school, the ratio of boys to girls is 5 is to 7. If there
measure of one of the base angles? are 180 boys and girls in the school, how many boys are there?
a. 150° a. 105
b. 60° b. 90
c. 75° c. 45
d. 80° d. 75
Answer: D Answer: D
13. Ana and Beth do a job together in three hours. Working alone, 19. Ruben’s grades in 6 subjects are 88, 90, 97, 90, 91 and 86? What
Ana does the job in 5 hours. How long will it take Beth to do the job is the least grade that he should aim for in the 7th subject if he has
alone? to have an average of 88?
a. 3 and 1/3 hours a. 92

23
b. 74 a. 84
c. 88 b. 7
d. 85 c. 12
Answer: B d. 14
20. On a certain day, three computer technicians took turns in Answer: C
manning a 24-hour internet shop. The number of hours Cesar, Bert, 25. Determine the midpoint of the line segment joining the points
and Danny were on duty was in the ratio 3:4:5, respectively. The (7, -3) and (-1, 6).
shop owner pays them P50 per hour. How much would Danny a. (2, 3/2)
receive for that day? b. (2, -3/2)
a. P 230 c. (3, 3/2)
b. P500 d. (1, 5/2)
c. P160 Answer: C
d. P480 26. Which of these has the longest perimeter?
Answer: B a. A square 21 cm on a side
21. A retailer buys candies for P90.25. The pack has 35 pieces of b. A rectangle 19 cm long and 24 cm wide
candies. If she sells each candy for P2.25, how much profit does she c. An equilateral triangle whose side is 28 cm
make? d. A right triangle whose two legs are 24 and 32 cm
a. P11.50 Answer: D
b. P56.25 27. How many square inches are in 2 square yard?
c. P37.50 a. 900
d. P18.75 b. 144
22. An online shop sells a certain calculator for P950 and charges c. 1296
P150 for shipping within Manila, regardless of the number of d. 2,592
calculators ordered. Which of the following equations shows the Answer: D
total cost (y) of an order as a function of the number of 28. In a playground for Kindergarten kids, 18 children are riding
calculators ordered (x)? tricycles or bicycles. If there are 43 wheels in all, how many
a. y = (950 + 150)x tricycles are there?
b. y = 150x +950 a. 8
c. x = 950y + 150 b. 9
d. y = 950x + 150 c. 7
Answer: D d. 11
23. One side of a 45° - 45° - 90° triangle measures x cm. What is the Answer: C
length of its hypotenuse? 29. Nelia takes ¾ hour to dress and get ready for school. It takes
a. X √3 cm 4/5 hour to reach the school. If her class starts promptly at 8:00
b. X cm am; what is the latest time she can jump out of bed in order not to
c. (X √3)/2 cm be late for school?
d. X √2 cm a. 6:42 am
Answer: D b. 6:27 am
24. The legs of one right triangle are 9 and 12, while those of c. 6:57 am
another right triangle are 12 and 16. How much longer is the d. 7:02 am
perimeter of the larger triangle than the perimeter of the smaller Answer: B
triangle? 30. Which common fraction is equivalent to 0.215?

24
a. 43/200 found to float over B and C. It was also found that liquid A flows
b. 27/125 fastest among the three. What can be said about liquid A?
c. 21/50 a. Densest and most viscous
d. 108/375 b. Densest and least viscous
Answer: A c. Least dense and most viscous
Below is the LET Reviewer for General Education GENED: Science d. Least dense and least viscous
Part 1. We encourage readers/ reviewees to use the comment boxes Answer: D
after the article for discussion. 6. Which of the following statements represents a physical change?
1. Which of the following statements best describes a hypothetical a. An antacid tablet forms bubbles when dissolved in water.
element with an electron configuration of 1s22s22p63s23p5? b. A flashlight beam slowly gets dimmer and finally dies out over
a. The hypothetical element has an atomic number of 11. time.
b. The hypothetical element is a member of Group V, otherwise c. The lawn grows thicker every day because fertilizers were added
called the Nitrogen Group. into the soil.
c. The hypothetical element is in the fifth position in the p-block, d. Frozen mango juice melted when left standing at room
along the third period of the periodic table. temperature for 30 minutes.
d. The hypothetical element is located at the third position of the p- Answer: D
block, along the fifth period of the periodic table. 7. Which of the following best explains why farmers burn rice straw
Answer: C and hull during seasons of harvest?
2. Despite the observed diversity among organisms, they are all a. Burning rice hulls and straws produce compounds that act as
made from the same set of biomolecules composed of monomeric repellant for pests which may damage plantation.
units except: b. The smoke produced by burning rice hulls and straws stimulate
a. Proteins growth and fruit bearing of trees.
b. Carbohydrates c. Rice hulls and straws are burned so that more spaces will be
c. Nucleic Acids available for planting next set of crops.
d. Lipids and Fats d. Ash from burnt rice hulls and straws are rich in compounds that
Answer: D could neutralize acidic soil so that more crops will grow
3. What will be formed when radium isotope, with 88 protons and 8. Acid rain occurs when ____________.
138 neutrons undergoes alpha decay? Answer: D
a. Radon Atom (Rn222) with 86 Protons 8. Acid rain occurs when _________.
b. Francium Atom (Fr222) with 87 Protons a. carbon dioxide combines with water in the atmosphere.
c. Actinium Atom (Ac222) with 89 Protons b. phosphorus-rich water in lakes evaporates to form phosphoric
d. Thorium Atom (Th232) with 90 Protons acid.
Answer: A c. sulfur released in burning fossil fuels combines with water in the
4. Which of the following examples best illustrates application of atmosphere.
Boyle’s Law? d. excess hydrogen is released into the atmosphere to produce
a. A tire becomes harder as more air is pumped into it. acids.
b. A sealed aerosol can explodes when thrown into a fire. Answer: C
c. A balloon expands and bursts when exposed to direct sunlight. 9. Why is it difficult to integrate nitrogen gas from the atmosphere
d. A scuba divers stops at certain depths as he ascends to the into the nitrogen cycle of the biosphere?
ocean’s surface. a. Nitrogen is very abundant in the atmosphere
Answer: D b. Living organisms quickly absorb nitrogen gas
5. Three liquids A, B, C were studied in a laboratory. Liquid A was c. Oceans quickly absorb nitrogen gas

25
d. Few organisms can directly utilize atmospheric nitrogen Grass>Grasshopper>Frog>Snake
Answer: D Bird>Snake
10. Which of the following sentences about greenhouse effect is a. Grass population would increase.
INCORRECT? b. Grasshopper population would increase.
a. Greenhouse gases trap heat in the atmosphere which are c. Bird and frog populations would increase.
returned to the earth’s surface. d. Grasshopper and bird populations would increase.
b. Greenhouse effect is important in maintaining the temperature of Answer: C
the earth. 15. Which of the following shows mechanical weathering of rocks?
c. Greenhouse effect is due to gases that absorb the green region of a. formation of caverns
light from the sun. b. acids dissolves rocks
d. Greenhouse effect increases the overall surface temperature of c. freezing water between rock particles
the earth. d. iron in rocks combine with oxygen
Answer: C Answer: C
11. When a gardener propagates a plant by taking cuttings, he 16. PAGASA announces the approach of the seasonal winds. The
plants his cutting in a well-watered soil in a plant pot. What is the familiar names used are Amihan and Habagat, internationally
most likely reason why he may then cover the plant and pot with a known as ________ and ________ respectively.
lightly perforated polythene bag? a. Northeast and southwest
a. To reduce the water demand of the cutting. b. Trade wind and easterlies
b. To decrease the rate of gaseous exchange by the plant. c. Southwest and northeast
c. To reduce the chance of attack by pests. d. Westerlies and easterlies
d. To protect the plant from cold weather. Answer: A
Answer: A 17. Fog is a cloud with its base at or very near the ground. The
12. It is a common observation that mushrooms thrive few days formation of fog generally occurs after the ground has lost heat by:
after lightning strikes. Which among the following biogeochemical a. Evaporation
cycles is involved in this process? b. Convection
a. Nitrogen cycle c. Conduction
b. Carbon-Oxygen cycle d. Radiation
c. Phosphorus cycle Answer: D
d. Sulfur cycle 18. Why do we see the sun rise in the east?
Answer: A a. The earth revolves eastward.
13. Which of the following factors contribute to an increase in b. The earth rotates from west to east.
human population? c. We are located in the 20th meridian.
I. Immigration II. Emigration d. On the globe, we are located in the east.
III. Natality IV. Mortality Answer: B
a. III and IV 19. If a voltage of 100 volts produces a current of 5 amperes in an
b. II and III electrical device, what is the resistance?
c. I only a. 95 Ohms
d. I and III b. 20 Ohms
Answer: D c. 105 Ohms
14. Shown below is a simple food web in a grassy community. The d. 500 Ohms
arrow symbol means eaten by. What would happen if all snakes are Answer: B
killed? 20. Which of the following best differentiates an earthquake's

26
intensity from its magnitude? a. Double concave
a. Intensity describes 'the depth from which the earthquake b. Convex
originated’ while magnitude refers to ‘the energy of the c. Plane
earthquake’. d. Concave
b. Intensity cannot be measured while magnitude can be measured Answer: B
using a seismograph. 26. Why do we hear thunder some seconds after seeing lightning?
c. Intensity refers to the strength of the quake while magnitude a. Light appears brighter in the sky.
refers to the degree of destruction it caused at the epicenter. b. Light travels faster than sound.
d. Intensity is a measure of how much damage an earthquake cause c. Sound travels 1.331 m/s
at the surface while magnitude is the strength of the quake. d. Sound is released later actually
Answer: D Answer: B
21. Comparing the speed of sound in liquids, gases, and solids, the 27. Water has a higher specific heat than iron. What does this
speed of sound is usually lowest in ____ and highest in ____. mean?
a. solids, gases a. Water is hotter than iron
b. gases, liquids b. Water heats more rapidly than iron
c. liquids, solids c. Water is more dense than iron
d. gases, solids d. Water heats more slowly than iron
Answer: D Answer: D
22. Which has a greater density, a lake full of water or a cupful of 28. If a colorblind man marries a woman who has normal vision and
water? no history of the disease, it is most probable that all of their :
a. The cup full of water a. daughters will be carriers
b. The lake full of water b. daughters will be colorblind
c. Not enough information c. sons will be carriers
d. They have the same density d. sons will be colorblind
Answer: D Answer: A
23. A stainless steel spoon feels colder than a plastic spoon because 29. Albino corn seedlings may grow several inches tall. However,
stainless steel they will eventually die, primarily because
a. absorbs less heat from the hand than plastic does a. direct sunlight will destroy their cells
b. is really colder than plastic b. they lack adequate root system
c. has a lower temperature than plastic c. they cannot produce their own food
d. conducts heat away from the hand faster than plastic does d. they cannot obtain carbon dioxide
Answer: D Answer: C
24. Why is it NOT advisable to repeatedly open the door of a 30. Mimosa pudica, locally known as Makahiya is called as such
refrigerator? because its leaflets tend to close when touched. This organismal
a. It will loosen the hinges of the refrigerator's door response to a given stimuli is called:
b. Leads to wastage in electrical energy. a. Hydrotropism
c. Repeated opening introduces bacteria in to the refrigerator. b.Thigmotropism
d. The warm air outside lowers the temperature inside thus making c. Geotropism
the refrigeration less-efficient. d. Phototropism
Answer: B Answer: B
25. What kind of mirror is used in cars to give the driver a wider Below is the LET Reviewer for General Education GENED: Social
area and smaller image of the traffic behind him/her? Science Part 1. We encourage readers/ reviewees to use the

27
comment boxes after the article for discussion. a. Lack of models among the very people expected to exemplify
1. What factor was a major cause of both World War I and World these values
War II? b. Use of approaches which are mainly cognitive rather than
a. The spread of Marxian ideas into Europe effective
b. The dropping of atomic bombs c. Lack of follow up systems from one grade level to another
c. Nationalism and national borders d. Minimum recognition and appreciation given to teachers
d. The rise of totalitarian fascist states Answer: A
Answer: D 7. To govern is to rule and the government rules by laws. Whose
2. In which organization is the Philippines a member to fight main duty is the enforcement of laws?
communist aggression? a. Police department
a. APEC b. Judiciary department
b. ASA c. Legislative department
c. UN d. Executive department
d. SEATO Answer: D
Answer: D 8. The term that refers to the class of Filipinos who were free and
3. The theory that population increases by geometrical ratio while independent
the means of subsistence increases by arithmetical ratio is a. Timawa
attributed to? b. Maharlika
a. Karl Marx c. Aliping namamahay
b. Robert Malthus d. Aliping saguiguilid
c. Emile Durkheim Answer: A
d. Aristotle 9. The Spanish expedition responsible for naming the archipelago
Answer: B Filipinas
4. In what instance is the Filipino double-standard morality shown? a. Magellan’s expedition
a. A couple brings their sick child to the doctor then later to the b. Loarca expedition
espiritista c. Legaspi expedition
b. Young parents bring up their children in a manner different from d. Villalobos expedition
how they were brought up Answer: D
c. Illiterate parents are eager to send their children to school even if 10. The third and last military governor of the Philippines was
they themselves did not go to school a. Gen. Wesley Merritt
d. A married man who flirts with someone else other than his wife b. Gen. Elwell Otis
seems acceptable but a married woman who flirts with another c. Gen. Arthur MacArthur
man is condemned. d. Gen. Douglas MacArthur
Answer: D Answer: C
5. Which part of Asia does the Arabian peninsula occupy? 11. Which Katipunan member commuted from Cavite to Manila to
a. Northwest buy materials used to make ammunitions?
b. Southeast a. Teresa Magbanua
c. Southwest b. Agueda Esteban
d. Northeast c. Teodora Alonso
Answer: C d. Trinidad Tecson
6. The important factors which have contributed to the weakness in Answer: B
the internalization of desirable values is the? 12.Which economic system is based on free enterprise?

28
a. Globalism c. Led a successful nationalistic movement in their respective
b. Mixed economies countries
c. Capitalism d. Supported Marxist philosophy to change existing governments
d. Communism Answer: B
Answer: C 19. Nebuchadnezzar was to the Babylonian Empire as Asoka was to
13. How is the so-called colonial mentality manifested? the ______ Empire.
a. Cultural relativism a. Roman
b. Cultural diversity b. Gupta
c. Xenocentrism c. Greek
d. Ethnocentrism d. Maurya
Answer: C Answer: D
14. Which is a safeguard against unfair trade practices like short- 20. With the opposition of the parity rights in mind, who does NOT
weighing? belong to the group?
a. Total Quality Movement a. Claro M. Recto
b. Consumerism b. Jose Laurel
c. Consumer vigilance c. Manuel Roxas
d. Substandardization d. Pedro Taruc
Answer: C Answer: C
15. If the seven continents were arranged from largest to smallest, 21. When the Filipino reformists asked for the assimilation of the
in which order does Australia fall? Philippines by Spain, what did they ask for? For the Philippines to
a. 4th ____
b. 5th a. Become independent from Spain
c. 6th b. Become a province of Spain
8. 7th c. Be independent from Spain with certain conditions
Answer: D d. Be represented in the Spanish Cortes
16. In which continent can we find stormy Cape Horn which is Answer: B
known as the graveyard of ships and sailors? 22. Who among the Presidents changed the date of our celebration
a. Africa of Independence day from July 4 to June 12?
b. Australia a. Ramon Magsaysay
c. South America b. Diosdado Macapagal
d. Asia c. Carlos Garcia
Answer: C d. Ferdinand Marcos
17. Which led to the creation of Pakistan as a nation in 1947? Answer: B
Religious differences between 23. In which country did the Philippines participate in the world’s
a. Hindus and Christians peacekeeping operations by sending doctors, nurses, soldiers and
b. Christians and Muslims police?
c. Hindus and Buddhists a. Israel
d. Hindus and Muslims b. East Timor
Answer: D c. Iraq
18. Which is common to Sun Yat Sen and Mahatma Gandhi? d. Iran
a. Promoted a society ruled by religious leaders Answer: C
b. Rejected violence as a way to political power 24. In which poem did Rizal write about offering one’s life for one’s

29
country? 30. The following are our constitutional rights EXCEPT to
a. A La Juventud Filipina a. Free access to legal assistance which shall not be denied due to
b. Song of Maria Clara poverty
c. Sa Aking mga Kabata b. Have access to all records of the government
d. Mi Ultimo Adios c. Be presumed innocent until proven guilty
Answer: D d. Form association and labor unions
25. As an effect of our geography, in which of the following island/s Answer: B
do people travel mostly by water? Below is the LET Reviewer for General Education GENED:
a. Luzon Information and Communication Technology (ICT) Part 1. We
b. Mindanao encourage readers/ reviewees to use the comment boxes after the
c. Visayas article for discussion.
d. Visayas and Mindanao 1. Which of the following is not a benefit of technology in
Answer: C education?
26. As an insular country, to which principle does the Philippines a. Improves critical-thinking abilities.
adhere when it comes to territorial boundary? b. Unlimited access to games and other internet sites.
a. Two hundred nautical miles of the country’s coast c. Allows cooperative learning.
b. Three hundred fifty nautical miles from shore d. Increases self-expression.
c. Three-mile territorial limit Answer: B
d. Archipelagic doctrine 2. Which situation shows that technology can be used to motivate
Answer: D students to learn?
27. Which part/s of the Visayas has/have comparatively more a. Teacher Eric employs educational games related to the lesson
excessively moist climate and limited arable lands? they are taking.
a. Western b. Miss Cleo designs lessons which use cooperative learning with
b. Eastern technology integration.
c. Eastern and Central c. Mr. Jimmy utilizes computer assisted instruction programs so
d. Central and Western that students will learn at their own pace.
Answer: B d. Teacher Sarah assigns projects to her class where self-expression
28. Which follows Pres. Garcia’s “Filipino First Policy”? Filipinos and creativity is acknowledged.
a. Should buy and consume Filipino products only Answer: A
b. Should not contribute to the brain drain problem 3. He is considered as the “Father of Modern Media in Education”.
c. Should be selective in the entry in the entry of foreign a. Jean Piaget
professionals in the country b. Robert Gagne
d. Were to be given first preference in all matters related to the c. B.F. Skinner
economic development of the country d. Edgar Dale
Answer: D Answer: D
29. Aside from the Philippines, which countries claim part of the 4. It is a term to denote a whole range of technologies associated
Spratly islands in the South China Sea? with processing information and with sending and receiving
a. China, Thailand, Brunei, Malaysia messages.
b. China, Taiwan, Vietnam, Malaysia a. Educational Technology
c. Indonesia, China, Taiwan, Malaysia b. Information and Communication Technology
d. China, Thailand, Brunei, Malaysia c. Media Technology
Answer: B d. Instructional Systems Design

30
Answer: B b. accuracy
5. A methodology widely used for developing new training c. coverage
programs. d. currency
a. Microsoft Answer: B
b. World Wide Web
c. Instructional Systems Design
d. Computer Technology PROFESSIONAL EDUCATION
Answer: C 50 Items with Answer
6. This is considered to be the first manual data processing device 1. Which of the following would be the most fitting action of a teacher
developed in China in the 12th century A.D. who is having a relationship with his/her student?
a. Hieroglyphics a. Deny the relationship.
b. Papyrus b. Enjoy the relationship while it lasts.
c. Printing Press c. Defer the relationship until they are ready to admit it.
d. Abacus d. Continue the relationship and exercise utmost professional discretion
Answer: D about this.
7. He is considered to be the “Father of Computing” because of his 2. Ms. de Leon is a sickly teacher. She gets to school even late and not
contributions to the basic design of computer. feeling well. What provision in R.A. 7836 does she violates?
a. John Napier a. Teachers must be devoted, honest and punctual in performing their
b. William Oughtred duty.
c. Blaise Pascal b. Teachers should be physical, mentally and morally fit to teach.
d. Charles Babbage c. Teachers should manifest genuine enthusiasm and pride in teaching
Answer: D as a noble profession.
8. Which is the standard input device that accepts letters, numbers d. All of the above
and commands from the user? 3. "Education is a continuous process of experiencing and of visiting or
a. Trackpad reorganizing experiences" according to a Progressivist. What does it
b. Lightpen mean?
c. Mouse a. Education begins and ends in school.
d. Keyboard b. Education takes place anytime and anywhere.
Answer: D c. Education happens formally and informally.
9. Mr. Rico carefully studies the materials he acquired from the d. Education goes on throughout life.
internet. He always examines if the author is qualified to present 4. If you have a foreign visitor, what would be the best gift to buy?
the material. This practice is? a. Imported antiques
a. necessary to ensure that the materials are reliable. b. Expensive Buddha figurines
b. unethical because he does not trust the author. c. Porcelain wares
c. unnecessary because all resources from the internet are d. Embroidered products from Quezon province
dependable. 5. Which of the following would manifest best the Teacher-Student
d. just a waste of time. Relationship?
Answer: A a. Teacher to inflict corporal punishment to offending student upon the
10. When a teacher asks the consistency of a material taken from approval of parents.
the internet with other available materials, he/she is concerned b. Teacher to inflict corporal punishment on offending student secretly.
with its? c. Teacher to inflict corporal punishment to offending student at all
a. relevance times.
d. Teacher to inflict no corporal punishment on offending student at all
31
times. c. Lao Tzu
6) Which of the following is a characteristic of a Pragmatist Teacher? d. Plato
a. Equipping the learners with the basic abilities and skills. 13) According to Max Scheler's Hierarchy of Values, ________ is the
b. Providing the learners with the opportunities to apply theories and highest form of values.
principles. a. Pleasure values
c. Requiring the learners the full mastery of the lessons. b. Vital values
d. Encourage the learners to memorize factual knowledge. c. Spiritual values
7) In line with the philosophy of Reconstructivism, which of the d. Values of the Holy.
following should be given emphasis in teaching? 14) Which of the following prepositions is attributed to Plato?
a. To seek for a better position in the society. a. Truth is relative to a particular time and place.
b. To compare oneself with the less fortunate. b. Human beings create their own truth.
c. To become economically self-reliant. c. Learning is the discovery of truth as latent ideas are brought to
d. To designate one's superiority over the others. consciousness.
8) One of the responsibilities of school administrators and faculty d. Sense perception is the most accurate guide to knowledge.
members is to communicate and cooperate harmoniously with the 15) If reality is politically socially and economically formed, this
community. Which of the following attributes best applies to this philosophy is being followed:
responsibility? a. Essentialism
a. Interfacing b. Existentialism
b. Alignment c. Progressivism
c. Partnership d. Pragmatism
d. Kinship 16) In Piaget's concrete operational stage, a teacher should provide
9) This principle was violated when Teacher Rey talked badly about the ________.
new school principal in front of his students by telling them that the a. Activities for hypothesis formulation.
new principal is incompetent and arrogant. b. Learning activities that involve problems of classification and order.
a. Respect for authority c. Activities for evaluation purposes.
b. Fidelity d. Stimulating environment with ample objects to play with.
c. Loyalty 17) A grade 1 pupil likes to play with his friends, but gets angry when
d. Confidentiality defeated. Piaget's theory states that this pupil is under what
10) Which of the following is an integral part of the teaching process? development stage?
a. Teacher and principal a. Concrete-operational stage
b. Lesson plan b. Sensorimotor stage
c. Classroom management c. Formal-operational stage
d. Rubrics d. Pre-operational stage
11) Which of the following is the Fundamental Moral Principle? 18) Fear of something that was caused by a painful experience in the
a. Stealing is wrong. past is an example of?
b. Health is wealth. a. Insight
c. Do good and avoid evil. b. Classical conditioning
d. Cheating is immoral. c. Operant conditioning
12) "Moral example has greater effect on pupils' discipline than laws d. Imitation
and punishment" is an advice for teachers from ________. 19) A teacher rewards a child for doing things correctly. This technique
a. Confucius is called:
b. Mohammed a. Conditioning

32
b. Fading b. Superego and ego
c. Chaining c. Id
d. Reinforcement d. Ego
20) A person who is friendly and has a capacity to make people laugh 27) What idea about age is NOT right?
possesses: a. Chronological age is different from biological and psychological age.
a. Naturalistic intelligence b. Chronological age, biological age, psychological age and social age are
b. Spatial intelligence related.
c. Intrapersonal intelligence c. Psychological age and social age are two different things.
d. Interpersonal intelligence d. Chronological age is the same as biological and psychological age.
21) A student with Attention Deficit Disorder exhibits: 28) ________ reflects the teacher's understanding of development as
a. Care for his/her personal things results of maturation and learning.
b. Impatient while waiting for his/her turn during games a. Patience when dealing with the slower ones.
c. Completes work before shifting to another b. Creativity with classroom strategies or task.
d. Excessively quiet c. Fairness when giving grades or school marks.
22) A child treats his friends highly aggressive. The reason behind this d. Cheerfulness and enthusiasm when discussing.
attitude is his past experiences with his father who is also highly 29) ________ is Erikson's, Piaget's and Freud's thought about play.
aggressive. This is demonstrated on what theory? a. Contribute to the child's mastery of his physical and social
a. Social cognitive theory environment.
b. Cognitive developmental theory b. Makes a child's life enjoyable that he will tend to hate school life later.
c. Operant conditioning c. Prepares a child for an excellent academic performance in formal
d. Classical conditioning schooling.
23) Based on Jung's psychological theory, a child who is shy and prefers d. Develops the child's highly competitive attitude because of the nature
to be alone falls under what classification? of play.
a. Extrovert 30) A student has been staring at a puzzle. He is figuring out how to
b. Ambivert solve it and suddenly, an idea flashed in the student's mind and
c. Paranoid excitedly he was able to learn how to solve the puzzle. This exemplifies
d. Introvert ________.
24) ________ puts emphasis on the importance of sensitive periods in a. Metacognition
development. b. Insight learning
a. Ecological theory c. Analytical learning
b. Social Cognitive theory d. Trial and error learning
c. Psychoanalytic theory 31) A teacher attempting to develop a student's metacognitive skills
d. Ethnological theory teaches the student to:
25) Contrary to Freud's concept that the primary motivation of human a. Recall past lesson
behavior is sexual in nature, Erikson's is ________ in nature. b. Visualize
a. Social c. Formulate hypothesis
b. Cultural d. Think about their thinking
c. Physical 32) An adolescent exhibits what common characteristics?
d. Biological a. Reasonable and secure
26) ________ is described by Freud as the component/s of personality b. Feels intense emotions and sense of disequilibrium
that is concerned with the idea of right and wrong. c. Slow but steady physical secure
a. Superego d. Passive and obedient

33
33) Instincts are under what mental dimension? b. Involve the whole class in setting rules of conduct in the whole class.
a. Ego and Superego c. Make a report to the parents about their children's misbehavior.
b. Ego d. Set the rules for the class to observe.
c. Id 41) What is an inappropriate method in teaching young children?
d. Superego a. Individual difference are expected and accepted
34) A child cannot do ________, according to Piaget's concrete operational b. Integrated teaching-learning
stage. c. Isolated skill development
a. Doing mentally what was just physically done. d. Positive guidance techniques
b. Reasoning applied to specific example. 42) If a teacher believes that a child's mind is TABULA RASA, then the
c. Classifying objects into different sets. teacher will most likely engage the students in ________ process for them
d. Imagining the steps necessary to complete an algebraic expressions. to learn.
35) What should teachers teach students to eradicate bullying among a. Sensory impressions
kids in school? b. Reasoning
a. Full development of talents c. Reflections
b. Athletic skills d. Metacognition
c. Respect for the dignity of persons 43) A child received a candy after correctly completing his task. The
d. Full development of physical power child always tries to complete all tasks correctly for him to have a candy
36) Education is the acquisition of the art and the utilization of once again. What is being shown in the situation?
knowledge. This statement means: a. Associative learning
a. A learner's application of what she has learned is necessary. b. Classical conditioning
b. A learner's interest in art is commendable. c. Operant conditioning
c. A learner's acquisition of information is sufficient. d. Pavlovian conditioning
d. A learner's acquisition of information is not important. 44) A child learns by association and by insights. This states that the
37) Student's initiative is stifles by: association and cognitive theories of learning are:
a. Rationalism a. Diametrically opposed
b. Extreme authoritarianism b. Complementary
c. "Utang na loob" c. Partly wrong
d. "Bahala na" d. Partly correct
38) A piece of music may sound sad, but when each note is played, there 45) Emotion's role on success and happiness is highlighted by Daniel
is nothing sad about it. This is based on the doctrine that says: Goleman's theory on emotional intelligence. How can the teacher best
a. The whole experience is equal to the sum of its parts. show empathy in the case of fighting students?
b. The whole experience is more than the sum of its parts. a. Reprimand the students so that other will not follow the misbehavior.
c. The whole experience is less than the sum of its parts. b. Tell the students to stop fighting so that there will be peace in the
d. The whole experience is not in any way related to the sum of its parts. classroom.
39) Which among the words below does NOT prevent the emergence of c. Make them realize how fighting negatively affects themselves and
truth which the learners and teachers are in search of? others.
a. Mindset d. Establish roles and responsibilities to avoid arguments among them.
b. Mask 46) The television program BATIBOT was developed because:
c. Open mind a. Children learn by conditioning.
d. Defense b. Children learn by discovery.
40) What a teacher should do if students are misbehaving in class? c. Children learn by trial and error.
a. Send the misbehaving pupils to the guidance counselor. d. Children learn by observing and imitating.

34
47) Comprehension skills comprises the following, EXCEPT for: C
a. Finding the main idea 46. D 47. B 48. A 49. A 50. A
b. Recreational reading
c. Sequencing events chronologically PROFESSIONAL EDUCATION (Secondary)
d. Reading critically
48) Who was responsible for stating that modeling is essential in MULTIPLE CHOICE
pedagogy?
a. Bandura 1. Which does Naom Chomsky assert about language learning for
b. Bruner children?
c. Skinner I. Young children learn and apply grammatical rules and
d. Thorndlike vocabulary as they are exposed to them.
49) To make her students participate more often during class II. Begin formal teaching of grammatical rules to children as
discussions, teacher Chai believes that she can apply Operant early as possible.
Conditioning by way of: III. Do not require initial formal language teaching for children.
a. Giving more incentives to her students, such as additional grades and
small tokens. A. I and III C. I only
b. Emphasizing group sharing and guiding them in thinking critically B. II only D. I and II
and innovatively.
c. Instilling metacognition in them through modeling. 2. Which is/are the sources of man’s intellectual drives, according to
d. Letting them reflect on how they think about their subjects. Freud
50) "Porma" over substance is evident in students in one of these A. Id C. Id -ego
instances. B. Super ego D. Ego
a. Dolly submitted his comprehensive written report to Mrs. Topacio
even if it was slightly soiled with tomato ketchup. 3. Theft of school equipment like TV, computer, etc. by teenagers in the
b. Harvey submitted his written report on time to Mrs. Francisco even if community itself is becoming a common phenomenon. What does this
it reeked of the cigarette she smoked before class. incident signify?
c. Joel submitted his written report to Mrs. Cantiveros a week in A. Prevalence of poverty in the community.
advance. B. Inability of school to hire security guards.
d. Olivia submitted her poorly written report, which was printed on C. Deprivation of Filipino schools.
perfumed stationery to her English teacher, Mrs. Estrada. D. Community’s lack of sense of co-ownership.

4. As a teacher, you are reconstructionist, Which among these will be


KEY TO CORRECTION your guiding principle?
1. D2. B 3. D 4. D 5. D 6. B 7. C 8. C 9. A A. I must teach the child every knowledge, skills and value that he
needs for a better future.
10. C 11. C 12. A 13. D 14. C 15. C 16. B 17. D 18. B. I must teach the child to develop his mental powers to the full.
B C. I must teach the child so he is assured of heaven.
19. D 20. D 21. B 22. A 23. D 24. C 25. A 26. A 27. D. I must teach the child that we can never have real knowledge of
D anything.
28. A 29. A 30. B 31. D 32. B 33. C 34. D 35. C 36.
A 5. The concepts of Trust vs. mistrust, autonomy vs. shame & self-doubt,
37. B 38. D 39. C 40. B 41. C 42. C 43. C 44. B 45. and initiative vs. guilt are most closely related with works of _____.
35
A. Erickson C. Freud 12. Principal B tells her teachers that training in the humanities is most
B. Piaget D. Jung important. To which educational philosophy does he adhere?
A. Existentialism C. Progressivism
6. Student Z does not study at all but when the Licensure Examination B. Perennialism D. Essentialism
for Teachers comes, before he takes the LET, he spends one hour or
more praying for a miracle, i.e., to pass the examination. Which 13. All subject in Philippine elementary and secondary schools are expected
attitude towards religion or God is displayed? to be taught using the integrated approach. This came about as a result
A. Religion as fake C. Religion as authentic of implementation of ___________.
B. Religion as magic D. Religion as real A. Program for Decentralized Education
B. School-Based Management
7. Teacher B engages her students with information for thorough C. Basic Education Curriculum
understanding, for meaning and for competent application. Which D. D. Schools First Initiative
principle governs Teacher B’s practice?
A. Constructivist C. Behaviorist 14. Principal C shares this thought with the teachers: Subject matter should
B. Gestalt D. Cognitivist help students understand and appreciate themselves as unique
individual who accept complete responsibility for their thoughts,
8. Student B claims: “I cannot see perfection but I long for it. So it must feelings and actions. “ From which philosophy is this thought based?
be real. “under which group can he be classified? A. Perennialism C. Existentialism
A. Idealist C. Realist B. Essentialism D. Progressivism
B. Empiricist D. Pragmatist
15. Based on Piaget’s theory, what should a teacher provide for children in
9. What does extreme authoritarianism in the home reinforce in the concrete operational stage?
learners A. Activities for hypothesis formulation.
A. Doing things on their own initiative B. Learning activities that involve problems of classification and
B. Ability to direct themselves ordering.
C. Dependence on others for directions. C. Games and other physical activities to develop motor skills.
D. Creativity in work. D. Stimulating environment with ample objectives to play with.

10. You arrive at knowledge by re-thinking of latent ideas. From whom 16. To come closer to the truth we need to “go back to the things
does this thought come? themselves.” This is the advice of the ______.
A. Experimentalist C. Idealist A. Behavioris C. idealist
B. Realist D. Existentialist B. phenomenologists D. pragmatists

11. Behavior followed by pleasant consequences will be strengthened 17. Researches conducted show that teacher’s expectations of students
and will be more likely to occur in the future. Behavior followed by become self-fulfilling prophecies. What is this phenomenon called?
unpleasant consequences will be weakened and will be more likely to A. Halo effect C. Ripple effect
be repeated in the future. Which one is explained B. Pygmalion effect D. Hawthorne effect
A. Freud’s psychoanalytic theory
B. Thorndike’s law effect 18. Teacher F is convinced that whatever a student performs a desired
C. B.F. Skinner’s Operant conditioning theory behavior, provided reinforcement and soon the student will learn to
D. D. Bandura’s social learning theory perform the behavior on his own. On which principle is Teacher F’s
conviction based?
A. Cognitivism C. Behaviorism
36
B. Environmentalism D. Constructivism D. National Scholarship Program

19. Teacher U teaches to his pupils that pleasure is not the highest good. 25. Which of the following prepositions is attributed to Plato?
Teacher’s teaching is against what philosophy? A. Truth is relative to a particular time and place.
A. Realism C. Epicureanism B. Human beings create their own truths.
B. Hedonism D. Empiricism C. Learning is the discovery of truth as latent ideas are brought to
consciousness
20. Studies in the areas of neurosciences discloses that the human brain D. Sense perception is the most accurate guide to knowledge.
has limitless capacity. What does this imply
A. Some pupils are admittedly not capable of learning. 26. In a treatment for alcoholism, Ramil was made to drink an alcoholic
B. Every pupil has his own native ability and his learning is limited to beverage and then made to ingest a drug that produces nausea.
this native ability. Eventually, h nauseated at he sight and smell of alcohol and stopped
C. Every child is a potential genius. drinking alcohol. Which theory explains this?
D. Pupil can possibly reach a point where they have learned A. Operant conditioning C. Associative learning
everything. B. Social learning theory D. Attribution theory

21. Availment of the Philippine Education Placement Test for adults and 27. In a social studies class, Teacher I present a morally ambiguous
out-of-school youths is in support of the government’s educational situation and asks his students what they would do. On whose theory
program towards _____. is Teacher I’s technique based
A. equitable access C. quality and relevance A. Kohlberg C. Piaget
B. quality D. relevance B. Bandura D. Bruner

22. A mother gives his boy his favorite snack every time the boy cleans up 28. Quiz to formative test while periodic is to _________.
his room. Afterwards, the boy cleaned his room everyday in A. criterion-reference test C. norm-reference test
anticipation of the snack. Which theory is illustrated? B. summative test D. diagnostic test
A. Associative learning
B. Classical conditioning 29. Your teacher is of the opinion that the world and everything in it are
C. Operant conditioning ever changing and so you the skill to cope with change. What is his
D. Pavlonian conditioning governing philosophy?
A. Idealism C. Experimentalism
23. Which group of philosophers maintain the “truth exists in an objective B. Existentialism D. Realism
order that is independent of the knower”?
A. Idealists C. Existentialists 30. The search for related literature by accessing several data bases by the
B. PragmatistsD. Realists use of a telephone line to connect a computer that have database is
termed ______.
24. Under which program were students who were not accommodated in A.compact disc search C. on-line search
public elementary and secondary schools because of lack of classroom, B. manual search D. computer search
teachers, and instructional materials, were enrolled in private schools 31. In which competency do my students find the greater difficulty? In
in their respective communities at the government’s expense? item with a difficulty index of
A. Government Assistance Program A. 0. 1 C. 0.5
B. Study Now-Pay later B. 0. 9 D. 1.0
C. Educational Service Contract System

37
32. We encounter people whose prayer goes like this: O God, if there is a A. Shintoism C. Confucianism
God; save my soul if I have a soul,” from whom is this prayer? B. Zen Buddhism D. Taoism
A. Stoic C. Agnostic
B. Empiricist D. Skeptic 40. Teacher Y does norm-referenced interpretation of scores. Which of the
following does she do?
33. If teacher wants to test student’s ability to organize ideas, which type A. She describes group performance in relation to a level of mastery
of test should she formulate? set.
A. Short answer C. Essay B. She uses a specified content as its frame of reference.
B. Technical problem type D. Multiple-choice type C. She compares every individual student’s scores with others’ scores.
D. She describes what should be their performance.
34. Who among the following needs less verbal counseling but needs
more concrete operational forms of assistance? The child who ________. 41. The best way for a guidance counselor to begin to develop study skills
A. has mental retardation and habits in underachieving student would be to ______.
B. has attention-deficit disorder A. have these underachieving students observe the study habits of
C. has learning disability excelling students.
D. has conduct disorder B. encourage students to talk study habits from their own experiences
C. have them view filmstrips about various study approaches
35. Which applies when skewness is zero? D. give out list of effective study approaches
A. Mean is greater than median.
B. Median is greater than the mean. 42. Teacher F wanted to teach the pupils the skills to do cross stitching. He
C. Scores have three modes. check up quiz was a written test on the steps of cross stitching. What
D. Scores are normally distributed characteristic of a good test does it lack?
A. Scorability C. Objectivity
36. Which does NOT belong to the group of alternative learning systems B. Reliability D. Validity
A. Multi-grade grouping C. Graded education
B. Multi-age grouping D. Non-graded grouping 43. For which may you use the direct instruction method?
A. Become aware of the pollutants around us.
37. The first thing to do in constructing a periodic test is for a teacher to B. Appreciate Milton’s Paradise Lost.
______. C. Use a microscope properly.
A. decide on the number of items for the test D. Distinguish war from aggression.
B. go back to her instructional objectives
C. study the content 44. In the context on the theory on multiple intelligences, what is one
D. decide on the type of test to construct weakness of the paper-pencil test?
A. It is not easy to administer.
38. Which one can enhance the comparability of grades? B. It puts the non-linguistically intelligent at a disadvantage
A. Using common conversation table for translating test scores in to C. It utilizes so much time.
ratings. D. It lacks reliability.
B. Formulating tests that vary from one teacher to another.
C. Allowing individual teachers to determine factors for rating 45. NSAT and NEAT results are interpreted against set mastery level. This
D. Individual teachers giving weights to factors considered for rating. means that NSAT and NEAT fall under _________.
A. intelligence test C. criterion-referenced test
39. The cultivation of reflective and meditative skills in teaching is an B. aptitude test D. norm-referenced test
influence of _____.
38
B. Term of Specifications D. Table of Specifications
46. Teacher B uses the direct instruction strategy. Which sequence of steps
will she follow? 52. Which is a major advantage of a curriculum-based assessment?
I. Independent practice A. It is informal nature.
II. Feedback and correctiveness B. It connects testing with teaching.
III. Guided student practice C. It tends to focus on anecdotal information on student progress.
IV. Presenting and structuring D. It is based on a norm-referenced measurement model.
V. Reviewing the previous day’s work
53. Teacher H gave first-grade class a page with a story in which picture
A. V-IV-III-II-I C. V-II-IV-III-I take the place of some words. Which method did she use?
B. III-II-IV-I-V D. I-V-II-III-IV A. The whole language approach
B. The Spaulding method
47. Which guideline must be observed in the use of prompting to shape the C. The rebus method
correct performance of your students? D. The language experience approach
A. Use the list intrusive prompt first.
B. Use all prompts available. 54. Out of 3 distracters in a multiple choice test items, namely B, C, and D,
C. Use the most intrusive prompt first. no pupil choice D as answer. This implies that D is ________.
D. Refrain form using prompts. A. an ineffective distracter
B. a vague distracter
48. What measure/s of central tendency do/es the number 16 represent in C. an effective distracter
the following data: 14,15,17,16,19,20,16,14,16? D. a plausible distracter
A. Mode C. Median
B. Mode and median D. Mean 55. The burnout malady gets worse if a teacher doesn’t intervene to change
whatever areas he or she in control. Which one can renew a teacher’s
49. Which holds true to standardized tests? enthusiasm?
A. They are used for comparative purposes. A. Stick to job C. Judge someone else as wrong
B. They are administered differently. B. Initiate changes in jobs D. Engage in self-pity
C. They are scored according to different standards.
D. They are used for assigned grades. 56. If teacher has to ask more higher-order questions, he has to ask more
____ questions.
50. Study this group of tests which was administered with the following A. closed C. concept
results, then answer the question. B. Fact D. convergent
Subject Mean SD Ronnel’s scrore
Math 56 10 43 57. What can be said of Peter who obtained a score of P75 in a Grammar
Physics 41 9 31 objective test?
English 80 16 109 A. His rating is 75 items in the test correctly.
B. He answered 75 % of the test items correctly.
In which subject(s) were the scores most homogenous? C. He answered 75.
A. Math C. Physics D. He performed better that 5% of his classmate.
B. English D. Physics and Math
51. In the parlance of test construction what does TOS mean? 58. I drew learners into several content areas and encouraged them to solve
A. Table of Specifics C. Table of Specific Test Items a complex question for interdisciplinary teaching. Which strategy did I
use?
39
A. Problem-centered learning B. has a lower validity.
B. Unit method C. has a positive discriminating power.
C. Reading-writing activity D. has a high reability.
D. D. Thematic instruction
65. Teacher E discussed how electricity flows through wires and what
59. Which guidance in test construction is NOT observed in this test item “ generates the electric charge. Then she gave the students wires, bulbs,
Jose Rizal wrote ____”. switches, and dry cells and told the class to create a circuit that will
A. The central problem should be packed in the stem. increase the brightness of each bulb. Which one best describes the
B. There must be only one correct answer. approach used?
C. Alternatives must grammatical parallelism. A. It used a taxonomy of basic thinking skills.
D. The alternative must be plausible. B. It was constructivist.
C. It helped students understand scientific methodology.
60. To elicit more students’ response, Teacher G. made use of covert D. None of the above
responses. Which one did she NOT do?
A. She had the students write their response privately. 66. Teacher W wants to review and check on the lesson of the previous day.
B. She showed the correct answers on the overhead after the students Which one will be most reliable?
have written their responses. A. Having students identify difficult homework problems.
C. She had the students write their responses privately then called B. Having students correct each other’s work.
each of them. C. Sampling the understanding of a few students.
D. She refrained from judging on the student’s responses. D. Explicitly reviewing the task-relevant information necessary for the
day’s lesson.
61. Direct instruction id a facts, rules, and actions as indirect instruction is
for _____ ,____, _____. 67. Shown a picture of children in sweaters inside the classrooms, the
A. hypotheses, verified data and conclusions students were asked this question: In what kind of climate do these
B. concepts, patterns and abstractions children live? This is a thought questions on ______.
C. concepts, processes and generalizations A. inferring C. applying
D. guesses, data and conclusions B. creating D. predicting

62. Which test has broad sampling of topics as strength? 68. Study this group of tests which was administered with the following
A. Objective test C. Essay test results, then answer the question.
B. Short answer test D. Problem test Subject Mean SD Ronnel’s scrore
. Math 56 10 43
63. Teacher T taught a lesson denoting ownership by means of Physics 41 9 31
possessives. He first introduced the rule, then gave examples, English 80 16 109
followed by class exercises, then back to the rule before he moved into
second rule. Which presenting technique did he use In which subject(s) did Ronnel perform best in relation to the
A. Combinatorial C. Part-whole group’s performance?
B. Comparative D. Sequential
A. Physics and Math C. Physics
64. In his second item analysis, Teacher H found out that more from the B. English D. Math
lower group got the test item # 6 correctly. This means that the test
item _____. 69. Which criterion should guide a teacher in the choice of instructional
A. has a negative discriminating power. devices?
40
A. Attractiveness C. Novelty A. Personality tests C. Paper-and-pencil test
B. Cost D. Appropriateness B. B. Performance tests D. Standardized tests
70. Study this group of tests which was administered with the following
results, then answer the question. 77. Which is one role play in the pre-school and early childhood years?
Subject Mean SD Ronnel’s scrore A. Develops competitive spirit.
Math 56 10 43 B. Separates reality from fantasy.
Physics 41 9 31 C. Increase imagination due to expanding knowledge and emotional
English 80 16 109 range.
D. Develops the upper and lower limbs.
In which subject(s) did Ronnel perform poorly in relation to the
group’s performance? 78. Teacher A discovered that his pupils are very good in dramatizing.
A. English C. Math Which tool must have helped him discover his pupils’ strength?
B. English and Math D. Physics A. Portfolio assessment C. Journal entry
B. Performance test D. Paper-and-pencil test
71. “What is most likely to happen to our economy when export
continuously surpasses import” is a thought question on ______. 79. Teacher M’s pupils are quite weak academically and his lesson is
A. creating C. relating cause-and-effect already far behind time table. How should Teacher M proceed with his
B. synthesizing D. predicting lesson?
A. Experientially C. Logically
72. Which method has been proven to be effective in courses that stress B. B. Inductively D. Deductively
acquisition of knowledge?
A. Socratic method C. Indirect instruction 81. I want to teach concepts, patterns and abstractions. Which method is
B. Cooperative learning D. Mastery learning most appropriate?
A. Indirect instruction C. Direct instruction
73. Which is the first step in planning an achievement test? B. Discovery D. Problem solving
A. Define the instructional objective.
B. Decide on the length of the test. 82. If your Licensure Examination for Teacher (LET) items sample
C. Select the type of test items to use. adequately the competencies listed in the syllabi, it can be said that
D. Build a table of specification. LET possesses ______ validity.
A. concurrent C. content
74. Which activity should a teacher have more for his students if he wants B. construct D. predictive
to develop logical-mathematical thinking?
A. Drama C. Problem solving 83. Read the following then answer the questions:
B. Choral reading D. Storytelling
TEACHER: IN WHAT WAYS OTHER THAN THE PERIODIC TABLE
75. Why should a teacher NOT use direct instruction all the time MIGHT WE PREDICT THE UNDISCOVERED ELEMENTS?
A. It requires much time. BOBBY: WE COULD GO TO THE MOON AND SEE IF THERE ARE
B. It requires use of many supplemental materials. SOME ELEMENTS THERE WE DON’T HAVE.
C. It is generally effective only in the teaching of concepts and BETTY: WE COULD DIG DOWN TO THE CENTER OF THE
abstractions. EARTH AND SEE IF WE FIND ANY OF THE MISSING
D. It reduces student’s engagement in learning. ELEMENTS.
RICKY: WE COULD STUDY DEBRIS FROM THE METEORITES-IF
76. Which are direct measures of competence? WE CAN FIND ANY.
41
TEACHER: THOSE ARE ALL GOOD ANSWERS. BUT WHAT IF 89. With synthesizing skills in mind, which has the highest diagnostic value?
THOSE EXCURSIONS TO THE MOON, TO THE CENTER A. Essay test C. Completion tes
OF THE EARTH OR TO FIND METEORITES WERE TOO B. Performance test D. Multiple choice test
COSTLY AND TIME CONSUMING? HOW MIGHT WE USE 90. How can you exhibit legitimate power on the first day of school?
THE ELEMENTS WE ALREADY HAVE HERE ON EARTH A. By making your students feel they are accepted for who they are.
TO FIND SOME NEW ONES? B. By informing them you are allowed to act in loco parents.
C. By making them realize the importance of good grades.
Question: The Teacher questions in the above exchange are examples of D. By making them feel you have mastery of subject matter.
____ questions.
A. Fact C. Direct 91. Based on Edgar Dale’s Cone of Experience, which activity is farthest
B. Concept D. Closed from the real thing?
A. View images C. Watch a demo
84. In Krathwoh’s taxonomy of objectives in the affective, which is most B. Attend exhibit D. Hear
authentic?
A. Characterization C. Responding 92. Which can effectively measure student’s awareness of values?
B. Valuing D. Organization A. Projective techniques C. Likert scales
B. Moral dilemma D. Anecdotal record
85. “In the light of the facts presented, what is most likely to happen
when…?” is a simple thought question on _____. 93. I combined several subject areas in order to focus on a single concept
A. inferring C. synthesizing for interdisciplinary teaching. Which strategy/method did I use?
B. generalizing D. justifying A. Problem-entered learning
B. Thematic instruction
86. The teacher’s first task in the selection of media in teaching is to C. Reading–writing activity
determine the D. Unit method
A. choice of the students
B. availability of the media 94. The test item ”Group the following items according to shape” is a
C. objectives of the lesson thought test item on _______.
D. technique to be used A. creating C. classifying
B. generating D. comparing
87. In self-directed learning, to what extent should a teacher’s “scaffolding”
be? 95. For maximum interaction, a teacher ought to avoid _____ questions.
A. To a degree the student needs it. A. informational C. leading
B. None, to force the student to learn by himself B. rhetorical D. divergent
C. To the maximum, in order to extend to the student all the help he
needs. 96. By what name is Socratic method also known ?
D. To the minimum, to spend up development of student’s sense of A. Mastery learning C. Morrison method
independence. B. Indirect instruction D. Questioning Method

88. Which is a form of direct instruction? 97. Which is an appropriate way to make manage off-task behavior?
A. Discovery process C. Programmed instruction A. Make eye contact.
B. B. Problem solving D. Inductive reasoning B. Stop your class activity to correct a child who is no longer on task.
C. Move closer to the child.

42
D. Redirect a child’s attention to task and check his progress to make D. The teacher is emphasizing reading and writing skills.
sure he is continuing to work
104. With specific details in mind, which one has (have) a stronger
98. Which one can best evaluate student’s attitudinal development? diagnostic value?
A. Essay test C. Observation A. Multiple choice test
B. Portfolio D. Short answer test B. Non-restricted essay test
C. Restricted essay test
99. What should a teacher do for students in his class who are on grade D. Restricted and non-restricted essay tests
level
A. Give them materials on their level and let them work at a pace that 105. Teacher B is a teacher of English as Second Language: she uses
is reasonable for them, trying to bring them up to a grade level. vocabulary cards, fill-in-the blank sentences, dictation and writing
B. Give them the same work as the other students, because they will exercises in teaching a lesson about grocery shopping. Based on this
absorb as much as they are capable of. information, which of the following is a valid conclusion?
C. Give them the same work as the other students, not much, so that A. The teacher is reinforcing learning by giving the same information
they won’t feel embarrassed. in a variety of methods.
D. Give them work on the level of the other students and work a little B. The teacher applying Bloom’s hierarchy of cognitive learning.
above the classmates level to challenge them. C. The teacher is wants to do less talk.
D. The teacher is emphasizing listening and speaking skills.
100. With-it-ness, according to Kourin, is one of the characteristics of an
effective classroom manager. Which phrase goes with it? 106. In a criterion-referenced testing, what must you do to ensure that your
A. Have hands that write fast. test is fair
B. Have eyes on the back of your hands. A. Make all of the questions true or false.
C. Have a mouth ready to speak. B. Ask each student to contribute one question.
D. Have minds packed with knowledge. C. Make twenty questions but ask the students to answer only ten of
their choice.
101. On whose philosophy was A.S.Neil Summerhill, one of the most D. Use objectives for the unit as guide in your test construction.
experimental schools based?
A. A. Rousseau C. Montessori 107. Read this question: “How will you present the layers of the earth to
B. B. Pestalozzi D. John Locke your class?” This is a question that _________.
A. directs
102. Which Filipino trait works against the shift in teacher’s role from B. leads the students to evaluate
teacher as a fountain of information to teacher as facilitator? C. assesses cognition
A. Authoritativeness C. Hiya D. probes creative thinking
B. Authoritarianism D. Pakikisama
108. In Krathwoh’s affective domain of objectives, which of the following is
103. Teacher A is a teacher of English as Second Language: she uses the lowest level of effective behavior?
vocabulary cards, fill-in-the blank sentences, dictation and writing A. Valuing C. Responding
exercises in teaching a lesson about grocery shopping. Based on this B. Characterization D. Organization
information, which of the following is a valid conclusion?
A. The teacher is applying Bloom’s hierarchy of cognitive learning. 109. Which is NOT a sound purpose for asking questions?
B. The teacher is teaching variety of ways because not all students A. To probe deeper after an answer is given
learn in the same manner. B. To discipline a bully in class
C. The teacher wants to make her teaching easier by having less talk.
43
C. To remind students of a procedure BETTY: WE COULD DIG DOWN TO THE CENTER OF THE EARTH
D. To encourage self-reflection AND SEE IF WE FIND ANY OF THE MISSING ELEMENTS.
RICKY: WE COULD STUDY DEBRIS FROM THE METEORITES- IF
110. It is not wise to laugh at a two-year old child when he utters bad word WE CAN FIND ANY.
because in his stage he is learning to __________. TEACHER:THOSE ARE ALL GOOD ANSWERS BUT WHAT IF THOSE
A. considered other’s views EXCURSIONS TO THE MOON, TO THE CENTER OF THE
B. distinguish right from wrong EARTH OR TO FIND METEORITES WERE TOO COSTLY AND
C. socialize TIME CONSUMING? HOW MIGHT WE USE THE ELEMENTS
D. distinguish sex differences WE ALREADY HAVE HERE ON EARTH TO FIND SOME NEW
ONES?
111. Research tells that teachers ask mostly content questions. Which of
the following terms does NOT refer to content questions? Question: which questioning strategy/ies does/do the exchange of
A. Closed C. Concept thoughts above illustrate?
B. Direct D. Convergent A. Funneling
B. Sowing and reaping
112. In mastery learning, the definition of an acceptable standard of C. Nose-dive
performance is called a D. Extending and lifting
A. behavior C. SMART
B. condition D. criterion measure 117. How can you exhibit referent power on the first day of school?
A. By making them feel you know what you are talking about.
113. Based on Edgar Dale’s Cone of Experience, which activity is farthest B. By telling them the importance of good grades.
from the real thing? C. By reminding your students your authority over them again and
A. Read C. View images again.
B. Hear D. Attend exhibit D. By giving your students a sense of belonging and acceptance.

114. After giving an input on a good paragraph, Teacher W asks her 118. A sixth grade twelve-year old boy comes from a dysfunctional family
students to rate a given paragraph along the elements of a good and has been abused and neglected. He has been to two orphanages
paragraph. The students’ task is in level of _________. and three different elementary schools. The student can decode on the
A. application C. evaluation second grade level, but he can comprehend orally material at the fourth
B. analysis D. synthesis or fifth grade level. The most probable cause/s of this student’s reading
problem is/are ______.
115. Which is one characteristic of an effective classroom management? A. emotional factors
A. It quickly and unobtrusively redirects misbehavior once it occurs. B. poor teaching
B. It teaches dependence on other for self-control. C. neurological factors
C. It respects cultural norms of a limited group students. D. immaturity
D. Strategies are simple enough to be used consistently
119. Which questioning practice promotes more class interaction?
116. Read the following then answer the questions: A. Asking the question before calling a student
TEACHER: IN WHAT WAYS OTHER THAN THE PERIODIC TABLE B. Focusing on divergent questions.
MIGHT WE PREDICT THE UNDISCOVERED ELEMENTS? C. Focusing on convergent questions.
BOBBY: WE COULD GO TO THE MOON AND SEE IF THERE ARE D. Asking rhetorical questions
SOME ELEMENTS THERE WE DON’T HAVE.

44
120. The following are sound specific purpose of questions EXCEPTS developmental years, according to Erickson’s theory of psychological
A. To call the attention of an inattentive student development?
B. To teach via student answers A. Autonomy C. Initiative
C. To stimulate learners to ask questions B. Trust D. Generativity
D. To arouse interest and curiosity
127. How can you exhibit expert power on the first day of school?
121. Which technique should a teacher use to encourage response if his A. By making them feel you know what you are talking about.
students do not respond to his question? B. By telling them the importance of good grades.
A. Ask a specific student to respond, state the question, and wait a C. By reminding then your students your authority over them again
response. and again.
B. Tell the class that it will have detention unless answers are D. By giving your students a sense of belonging and acceptance.
forthcoming.
C. Ask another question, an easier one. 128. Teacher P wants to develop the skill of synthesizing in her pupil. Which
D. Wait for a response. one will she do?
A. Ask her students to formulate a generalization from the data shown
122. Which types of play is most characteristic of a four to six-year old in graphs.
child? B. Ask her students to answer questions beginning with “What if …”
A. Solidarity plays and onlookers plays C. Tell her pupils to state data presented in graphs.
B. Associative and cooperative plays D. Directs her students to ask questions on the parts of the lesson not
C. Associative and onlookers plays understood.
D. Cooperative and solidarity plays
129. John Watson said “Men are built not born. ” What does this statement
123. The principle of the individual difference requires teachers to point to?
___________. A. The ineffectiveness of training on a person’s development.
A. give greater attention to gifted learners B. The effect of environmental stimulation on a person’s development.
B. provide for a variety of learning activities C. The absence of genetic influence on a person’s development.
C. treat all learners alike while in the classroom D. The effect of heredity.
D. prepare modules for slow learners in class
124. Referring to Teacher S, Nicolle describes her teacher as “fair, caring 130. Teacher H strives to draw participation of every student into
and someone you can talk to”. Which power or leadership does Teacher S classroom discussion. Which student’s need is she trying to address?
have? The need
A. Referent power C. Reward power A. to show their oral abilities to the rest of the class
B. Legitimate power D. Expert power B. to be creative
C. to feel significant and be part of a group
125. During the Spanish period, what was/were the medium/media of D. to get everything out in the open
instruction in schools?
A. The Vernacular C. Spanish 131. Teacher G’s lessons objective has something to do with the skill of
B. English D. Spanish and the Vernacular synthesizing? Which behavioral term is most appropriate?
A. Test C. Appraisal
126. Rodel is very aloof and cold in his relationship with classmates. Which B. Assess D. Theorize
basic goals must have not been attained by Rodel during his
132. Which is a sound classroom management practice?
A. Avoid establishing routines; routines make your student robots.
45
B. Establish routines for all daily needs and tasks. C. Withdrawal of privileges
C. Apply rules and policies on a case to case basis. D. Raising the pitch of the voice
D. Apply reactive approach to discipline.
139. The primary objective of my lesson is: “To add similar fractions
133. A child who gets punished for stealing candy may not steal again correctly.” Before I can do this I must first aim at this specific
immediately. But this does not mean that the child may not steal again. objective: “to distinguish a numerator from a nominator. “What kind
Based on Thorndike’s theory on punishment and learning, this shows that of objective is the latter?
______________. A. Major C. Enabling
A. punishment strengthen e a response B. Terminal D. Primary
B. punishment remove a response
C. punishment does not remove a response 140. With which goals of educational institutions as provided for by the
D. punishment weakens a response Constitution is the development of work skills aligned?
A. To develop moral character
134. Which assumption underlines the teacher’s use of performance B. To teach the duties of citizenship
objectives? C. To inculcate love of country
A. Not every form of learning id observable. D. To develop vocational efficiency
B. Performance objectives assure the learner of learning.
141. In instructional planning it is necessary that parts of the plan from the
C. Learning is defined as a change in the learner’s observable first to the last have ___________.
performance. A. clarity C. coherence
D. The success of the learner is based on teacher’s performance. B. symmetry D. conciseness

135. As a teacher, what do you do when you engage yourself in major task 142. All of the following describe the development of children aged eleven
analysis? to thirteen EXCEPT
A. they shift from impulsivity to adaptive ability
A. Test if learning reached higher level thinking skills B. sex differences in IQ becomes more evident
B. Breakdown a complex task into sub-skills C. they exhibit increased objectivity in thinking
C. Determine the level of thinking involved D. they show abstract thinking and judgement
D. Revise lesson objectives
143. If a teacher plans a constructivist lesson, what will he most likely do?
136. The following are used in writing performance objectives EXCEPT Plan how he can _______.
A. delineate C. integrate A. do evaluate his student’s work
B. diagram D. comprehend B. do reciprocal teaching
C. do lecture to his students
137. Teacher B clears his throat to communicate disapproval of a student’s D. engage his students in convergent thinking
behavior. Which specific influence technique is this?
A. Signal interference C. Interest boosting 144. In mastery learning, the definition of an acceptable standard of
B. Direct appeal D. Proximity control performance is called a
A. SMART C. behavior
138. An effective classroom manager uses low-profile classroom control. B. criterion measure D. condition
What is a low-profile classroom technique?
A. Note to parents 145. “A stitch on time saves nine”, so goes the adage. Applied to classroom
B. After-school detention management, this means that we _______.
46
A. may not occupy ourselves with disruptions which are worth B. evaluation
ignoring because they are minor C. identification of topics
B. must be reactive in our approach to discipline D. formulation of objectives
C. have to resolve minor disruptions before they are out of control
D. may apply 9 rules out of 10 consistently 152. Based on Freud’s psychoanalytic theory which component (s) of
personality is (are) concerned with a sense of right and wrong?
146. Ruben is very attached to his mother and Ruth to her father. In what
developmental stage are they according to Freudian psychological A. Super ego C. Id
theory? B. Super-ego and Ego D. Ego
A. Oedipal stage C. Anal Stage
B. Latent stage D. Pre-genital stage 153. Who among the following puts more emphasis on core requirements,
longer school day, longer academic year and more challenging
147. What was the prominent educational issues of the mid 1980’s? textbooks?
A. Bilingual Education C. Accountability A. Perennialist C. Progressivist
B. Value Education D. Mainstreaming B. Essentialist D. Existentialist
148. Which behavior is exhibited by a student who is strong in
interpersonal intelligence? 154. A student passes a research report poorly written but ornately
A. Works on his/her own. presented in a folder to make up for the poor quality of the book report
B. Keeps interest to himself/herself content. Which Filipino trait does this practice prove?
C. Seeks out a classmate for help when problem occurs. A. art of academics
D. Spends time meditating. B. substance over “porma”
C. art over science
149. Which is behavioral term describes a lesson outcome in the highest D. “porma” over substance
level of Bloom’s cognitive domain?
A. Create C. 155. Which one may support equitable access but may sacrifice quality?
A. Open admission C. Deregulated tuition fee hike
B. School accreditation D. Selective retention

156. Based on Piaget’s theory, what should a teacher provide for children in
the sensimotor stage?
C. Analyze A. Games and other physical activities to develop motor skill.
B. Evaluate D. Design B. Learning activities that involve problems of classification and
ordering.
150. The main purpose of compulsory study of the Constitution is to ___. C. Activities for hypothesis formulation.
A. develop students into responsible, thinking citizens D. Stimulating environment with ample objects to play with.
B. acquaint student with the historical development of the Phil
Constitution 157. A teacher’s summary of a lesson serves the following functions,
C. make constitutional experts of the students EXCEPT
D. prepare students for law-making A. it links the parts of the lesson.
B. it brings together the information that has been discussed.
151. A goal-oriented instruction culminates in _______. C. it makes provisions for full participation of students.
A. planning activities D. it clinches the basic ideas or concepts of the lesson.

47
158. As a teacher, you are rationalist, Which among these will be your A. Submit a signed justifiable criticism against Teacher B, if there is
guiding principle? any.
A. I must teach the child that we can never have real knowledge of B. Go straight to the School Division Superintendent and gives
anything. criticism verbally.
B. I must teach the child to develop his mental powers to the full. C. Hire a group to distribute poison letters against Teacher B for the
C. I must teach the child so he is assured of heaven information dissemination.
D. I must teach the child every knowledge, skills and value that he D. Instigate student activists to read poison letter over the
needs for a better future. microphone.

159. Bruner’s theory on intellectual development moves from enactive to 166. Each teacher is said to be a trustee of the cultural and educational
iconic and symbolic stages. In which stage(s) are diagrams helpful to heritage of the nation and is under obligation to transmit to learners
accompany verbal information? such heritage. Which practice makes him fulfill such obligation?
A. Enactive and iconic C. Symbolic and enactive A. Use the latest instruction technology
B. Symbolic D. Iconic B. Observing continuing professional education.
160. Student’s scores on a test were: 72,72,73,74,76,78,81,83,85. C. Use interactive teaching strategies
The score 76 is the ___. D. Study the life of Filipino heroes
A. mode C. average
B. mean D. median 167. Which type of reports to “on-the spot” description of some incident,
episode or occurrence that is being observed and recorded as being of
161. Standard deviation is to variability as mode to _____. possible significance?
A. level of difficulty C. correction A. Autobiographical report
B. discrimination D. central tendency B. Value and interest report
C. Biographical report
162. A teacher who equates authority with power does NOT usually D. Anecdotal report
__________.
A. Shame C. develop self-respect in every pupil 168. Teacher A is directed to pass an undeserving student with a death
B. retaliate D. intimidate threat. Which advice will a hedonist give?
A. Pass the student. Why suffer the threat?
163. Which is a true foundation of the social order? B. Don’t pass him. You surely will not like someone to give you a
A. Obedient citizenry death threat in order to pass.
B. The reciprocation of rights and duties C. Pass the student. That will be use to the student, his parents and
C. Strong political leadership you.
D. Equitable distribution of wealth D. D. Don’t pass him. Live by principle of justice. You will get reward,
if not in this life, in the next.
164. Standard deviation is to variability as mean is to _______.
A.coefficient of correlation C. discrimination index 169. If you agree with Rizal on how you can contribute to our nation’s
B.central tendency D. level of difficulty redemption, which should you work for?
A. Opening our doors to foreign influence
165. Teacher Q does not want Teacher B to be promoted and so writes an B. Upgrading the quality of the Filipino through education
anonymous letter against Teacher B accusing her of fabricated lies. C. Stabilizing the political situation
Teacher Q mails this anonymous letter to School Division D. Gaining economic recovery
Superintendent. What should Teacher Q do if she has to act
professionally?
48
170. “All men are pretty much alike. It is only by custom that they are set B. No. This may exert undue influence on the members of the
apart, “ said one Oriental philosopher. Where can this thought be promotional staff and so may fail to promote on the basis of merit.
most inspiring? C. Yes. The rare invitation will certainly be welcomed by an
A. In a multi-cultural group of learners. overworked promotional staff.
B. In multi-cultural and heterogeneous groups of learners and D. Yes. There’s nothing wrong with sharing one’s blessings.
indigenous peoples’ group.
C. In a class composed of indigenous people. 177. Rights and duties are correlative. This means that.
D. In heterogeneous class of learners. A. rights and duties regulate the relationship of men in society.
B. rights and duties arise from natural law.
171. In what way can teachers uphold the highest possible standard of C. each right carries with it one or several corresponding duties.
quality education? D. rights and duties ultimately come from God.
A. By continually improving themselves personally and professionally
B. By wearing expensive clothes to change people’s poor perception 178. In the Preamble of Code of Ethics of Professional Teachers, which is
of teachers NOT said of teachers?
C. By working out undeserved promotions A. LET passer
D. By putting down other professions to lift the status of teaching B. Duly licensed professionals
C. Possess dignity and reputation
172. How would you select the most fit in government position? Applying D. With high moral values as well as technical and professional
Confucius teaching, which would be the answer? competence
A. By course accreditation of an accrediting body
B. By merit system and course accreditation 179. What should you do if a parent who is concerned about a grade his
C. By merit system child received compared to another student’s grade, demands to see
D. By government examinations both students’ grades?
A. Refuse to show either record.
173. The attention to the development of a deep respect and affection for B. Show only his child’s record.
our rich cultural past is an influence of ______ C. Refuse to show any record without expressing permission from
A. Confucius C. Teilhard de Chardin principal.
B. Heqel D. Dewey D. Show both records to him.

174. A teacher / student is held responsible for his actions because s/he 180. Teacher often complain of numerous non-teaching assignments that
_______. adversely affect their teaching. Does this mean that teachers must be
A. has instincts C. has mature preoccupied only with teaching?
B. has a choice D. is reason A. Yes, if they are given other assignments, justice demands that they
be properly compensated.
175. The typical autocratic teacher consistently does the following EXCEPT B. Yes, because other community leaders, not leaders, not teachers,
A. intimidating students C. shaming students are asked to lead in community activities.
B. ridiculing students D. encouraging students C. Yes, because every teacher is expected to provide leadership and
initiative in activities for betterment of communities.
176. Teacher H and teacher I are rivals for promotion. To again the favor of D. No, because teaching is enough full time job.
the promotion staff, teacher I offers her beach resort for free for
members of he promotional staff before ranking. As one of the 181. Which illustrates a development approach in guidance and counseling?
contenders for promotions, is this becoming of her to do A. Spotting on students in need of guidance
A. Yes. This will be professional growth for the promotional staff. B. Teaching students how to interact in a positive manner
49
C. Acting as a mentor
D. Making the decision for the confused student 188. A guest in one graduation rites told his audience: “Reminder, you are
what you choose to be”. The guest speaker is more of a/an _____.
182. Whose influence is the education program that puts emphasis on self- A. realistic C. pragmatist
development through the classics, music, and ritual? B. Idealistic D. existentialist
A. Buddha C. Confucius
B. Mohammed D. Lao tsu 189. From whom do we owe the theory of deductive interference as
illustrated in syllogism?
183. Helping in the development of graduates who are “maka-Diyos” is an A. Plato C. Socrates
influence of ___. B. Aristotle D. Pythagoras
A. naturalistic morality
B. classical Christian morality 190. In what way can teachers uphold the highest possible standard of
C. situational morality quality education?
D. dialectical morality A. By continually improving themselves personally and
professionally
184. What is the mean of this score distribution 4,5,6,7,8,9,10? B. By putting down other professions to lift the status of teaching
A. 7 C. 8.5 C. By wearing expensive clothes to change people’s poor perception
B. B. 6 D. 7.5 of teachers
D. By working out undeserved promotions
185. Are percentile ranks the same as percentage correct?
A. It cannot be determined unless scores are given. 191. Two students are given the WISC III. One has a full scale IQ of 91,
B. It cannot be determined unless the number of examinees is given while the other has an IQ of 109. Which conclusion can be drawn?
C. No A. Both students are functioning in the average range of intellectual
D. Yes ability
B. Another IQ test should be given to truly assess their intellectual
186. Teacher F is a newly converted to a religion. Deeply convinced of his potential.
new found religion, he starts Monday classes by attacking one religion C. The first student is probably below average, while the second has
and convinces his pupil to attend their religion services on Sundays. Is above average potential.
this in accordance with the Code of Ethics of Professional Teachers? D. The second student has significantly higher intellectual ability.
A. Yes. What he does strengthens value education.
B. No. A teacher should not use his position to proselyte others. 192. Which describes norm-referenced grading?
C. Yes. In the name of academic freedom, a teacher can decide what to A. What constitutes a perfect score
teach. B. The student’s past performance
D. Yes. What he does is a value education. C. An absolute standard
D. The performance of the group
187. In a study conducted, the pupils were asked which nationality they
preferred. If given a choice. Majority of the pupils wanted to 193. Teacher A knows of the illegal activities of a neighbor but keeps quiet
Americans. In this case, in which obligation, relative to the state, do in order not to be involved in any investigation. Which foundational
schools seem to be failing? In their obligation to principle of morality does Teacher A fail to apply?
A. respect for all duly constituted authorities A. Always do what is right
B. promote national pride B. The principle of double effect
C. promote obedience to the laws of the state C. The end does not justify the means
D. install allegiance to the Constitution
50
D. Between two evils, do the lesser evil B. formulation of objectives
C. identification of topics
194. Teacher Q does not want Teacher B to be promoted and so writes an D. planning activities
anonymous letter against Teacher B accusing her of fabricated lies.
Teacher Q mails this anonymous letter to School Division 200. Which of the following is considered a peripheral device?
Superintendent. What should Teacher Q do if she has to act A. Keyboard C. Monitor
professionally? B. CPU D. Printer
A. Hire a group to distribute poison letters against Teacher B for the
information dissemination.
B. Submit a signed justifiable criticism against Teacher B, if there is ANSWER KEY
any. PROFESSIONAL EDUCATION
C. Go straight to the School Division Superintendent and gives (Secondary)
criticism verbally.
D. Instigate student activists to read poison letter over the
microphone 1. A 50. C 100. D 150. B
197. D
195. A teacher’s summary of a lesson serves the following functions, 2. B 51. B 101. no answer 151. D
EXCEPT 198. C 3. D 52. D
A. it makes provisions for full participation of students. 102. B 152. A 199. A
B. it brings together the information that has been discussed. 4. A 53. no answer 103. B 153. no answer
C. it links the parts of the lesson. 200. D
D. it. clinches the basic ideas or concepts of the lesson 5. A 54. A 104. B 154. D
6. B 55. B 105. A 155. A
196. Soc exhibits fear response to freely roaming dogs but does not show 7. D 56. C 106. D 156. A
fear when a dog is on a leash or confined to a pen. Which 8. C 57. A 107. D 157 C
conditioning process is illustrated? 9. C 58. D 108. C 158. B
A. Extinction C. Generation 10. C 59. B 109. B 159. no answer
B. Discrimination D. Acquisition 11. C 60. D 110. C 160. D
12. D 61. B 111. D 161. D
197. Ruben is very attached to his mother and Ruth to her father. In what 13. C 62. C 112. C 162. C
developmental stage are they according to Freudian psychological 14. C 63. D 113. C 163. D
theory? 15. B 64. A 114. A 164. B
A. Latent stage C. Anal Stage 16. C 65. D 115. D 165. A
B. Pre-genital stage D. Oedipal stage 17. B 66. A 116. A 166. D
18. C 67. A 117. A 167. D
198. The concepts of Trust vs. mistrust, autonomy vs. shame & self-doubt, 19. B 68. C 118. A 168. D
and initiative vs. guilt are most closely related with works of _____. 20. C 69. D 119. B 169. B
A. Jung C. Erickson 21. A 70. A 120. A 170 B
B. Freud D. Piaget 22. B 71. C 121. C 171. A
23. A 72. C 122. C 172. D
199. A goal-oriented instruction culminates in _______. 24. C 73. A 123. B 173. A
A. evaluation 25. no answer 74. A 124. A 174. B

51
26. C 75. D 125. C 175. A
27. A 76. B 126. B 176. B
28. D 77. C 127. D 177. C
29. C 78. B 128. A 178. A
30. C 79. B 129. B 179. D
31. A 80. C 130. C 180. C
32. C 81. C 131. D 181. A
33. C 82. C 132. C 182. A
34. A 83. C 133. D 183. B
35. D 84. D 134. C 184. A
36. C 85. A 135. B 185. C
37. B 86. C 136. D 186. B
38. A 87. C 137. B 187. B
39. B 88. A 138. C 188. D
40 C 89. A 139. C 189. B
41. B 90. B 140. D 190. A
42. D 91. B 141. C 191. B
43. A 92. B 142. D 192. D
44. B 93. B 143. D 193. C
45 C 94. C 144. B 194. B
46. C 95. B 145. C 195. D
47. A 96. D 146. A 196. B
48. A 97. D 147. B
49. A 98. B 148. C
99. A 149. B

52
53

You might also like